Contoh Soal Baru

94
SOAL TRY OUT UNIBRAW JANUARI 2008 1. Male, 67yo complain of sudeen onset of decreased vision, metamorphosia, and paracentral scotoma. Clinically, we found elevation of the RPE, intraretinal fluid, subretinal blood, cystoid macular edema, and graygreen CNV elevation. The possible dx: a. NPDR+CSME b. Retinal angiomatous proliferation c. Retinal macroaneurisma d. Neovascular AMD. e. Polypoidal Choroidal Vasculopathy 2. If in the patient (no1) there is intraretinal haemorrhage at macular area, it may be an early sign of : a. PDR b. Retinal angiomatous proliferation. c. Macular BRVO d. Neovascular AMD e. Polypoidal Choroidal Vasculopathy 3. If in case no 1also found serosanguinous RPE detachment with multiple orange nodule, the possible dx is : a. Retinal angiomatous proliferation b. Neovascular AMD c. CSCR d. VKH syndrome e. Polypoidal Choroidal Vasculopathy. 4. For decision making process in the previous case (no 2&3), the gold standard is : a. fundus photography b. FFA c. ICG. d. OCT e. Multifocal ERG 5. Pria 43 tahun, dikonsulkan oleh dokter kulit karena akan memulai terpi klorokuin. Status oftalmologi yang harus diperiksa selain penampakan makula adalah : a. Tes penglihatan warna, amsler grid b. Tes penglihatan warna, amsler grid, fundus fotografi.

description

Contoh Soal Ophthalmology

Transcript of Contoh Soal Baru

Page 1: Contoh Soal Baru

SOAL TRY OUT UNIBRAW JANUARI 2008

1. Male, 67yo complain of sudeen onset of decreased vision, metamorphosia, and paracentral scotoma. Clinically, we found elevation of the RPE, intraretinal fluid, subretinal blood, cystoid macular edema, and graygreen CNV elevation. The possible dx:

a. NPDR+CSMEb. Retinal angiomatous proliferationc. Retinal macroaneurismad. Neovascular AMD.e. Polypoidal Choroidal Vasculopathy

2. If in the patient (no1) there is intraretinal haemorrhage at macular area, it may be an early sign of :a. PDRb. Retinal angiomatous proliferation.c. Macular BRVOd. Neovascular AMDe. Polypoidal Choroidal Vasculopathy

3. If in case no 1also found serosanguinous RPE detachment with multiple orange nodule, the possible dx is :

a. Retinal angiomatous proliferationb. Neovascular AMDc. CSCRd. VKH syndromee. Polypoidal Choroidal Vasculopathy.

4. For decision making process in the previous case (no 2&3), the gold standard is :a. fundus photographyb. FFAc. ICG.d. OCTe. Multifocal ERG

5. Pria 43 tahun, dikonsulkan oleh dokter kulit karena akan memulai terpi klorokuin. Status oftalmologi yang harus diperiksa selain penampakan makula adalah :

a. Tes penglihatan warna, amsler gridb. Tes penglihatan warna, amsler grid, fundus fotografi.c. Tes penglihatan warna, amsler grid, fundus fotografi, static perimetri.d. Tes penglihatan warna, amsler grid, static perimetrie. Tes penglihatan warna, amsler grid, fundus fotografi, ERG

6. Laki2 usia 62 tahun, mengeluh penglihatan mata kirinya mendadak turun sejak 12 hari yl, tanpa adanya nyeri. Visus OD 6/6, segmen anterior dalam batas normal, pada OS didapatkan gambaran n.optikus batas kabur dengan elevasi 2 D, hiperemi, perdarahan peripapil flameshaped. Pemeriksaan penunjang apakah yang diperlukan :

a. Automated perimetri, head MRI, konsul gigi mulut.b. Automated perimetri, head CT scanc. Goldman perimetri, konsul gigi mulutd. Goldman perimetri, gula darah puasa dan postprandiale. Konfrontasi tes dan head MRI

Page 2: Contoh Soal Baru

7. Bila didapatkan skotoma sekosentral pada hasil pemeriksaan lapang pandang mata kiri, maka pemeriksaan yang diperlukan sebelum melakukan terapi adalah :

a. Profil lipid dan hemoreologib. MRAc. Foto thorak AP laterald. Gula darah puasa dan postprandial.e. Analisa CSF

8. Wanita 35 tahun dengan obesitas mengeluh nyeri kepala yang berat, kadang melihat dobel, tajam penglihatan kedua mata 6/6. pernyataan yang sesuai adalah :

a. Pungsi lumbar harus segera dilakukanb. Penglihatan dobel terjadi karena keterlibatan N III, IV, VIc. Keadaan spt ini bisa terjadi pada kehamilan, obesitas, pemakaian tetrasiklin,

kontrasepsi oral.d. Kelainan lap pandang yang didapt berupa skotoma parasentrale. Tidak didapatkan kelainan reflek pupil

9. A patient come to clinic with severe panuveitis, the previouslyVA is 20/20 and now hand movement. The condition is not improve with cortosteroid treatment. The most appropriate additional treatment which properly given to this patient is :

a. Cyclophosphamideb. Chlorambucilc. Methotrexate.d. Azathioprinee. Cyclosporine

10. A patient 35 yo, presents with bilateral redness, photophobia, discomfort of the eye and floater. Anterior chamber cell and pars plana snowbank was found in examination. Periferal retinal phlebitis with retinal venous sheating wasa also observed. Which of the following statement is not true for the disease :

a. Assosiated with HLA DR2b. Often bilateral, but sometimes can be unilateralc. Major cause of blindness is abltio retina.d. Usually the cause is unknowne. More than 50% is recurrent

11. A patient presents with retinal haemorage, microaneurisma, and cotton wool spot on the retina. Blood examination shows CD4 count is 40 cells/mm3. Which of the following is not correct :

a. The retinal infection is commonly caused by CMVb. Cotton wool spot are the result of microvasculopathyc. Very early CMV retinitis commonly resmble as retinal haemorrhage.d. Ganciclovir is one of drug of choicee. Ganciclovir can be given orally

12. After underwent cataract surgery, patients visits the clinic with decrease vision, keartic precipitate, minimal hypopion and lens fragmen is found at antyerior chamber. Which of the following is not correct for the disease :

a. The inflammation is caused by autoimmune reaction to the lens proteinb. The reaction to the lens protein always granulomatous type.

Page 3: Contoh Soal Baru

c. The treatment is topical and systemic steroidd. Surgical removal of the lens is usually curativee. Intraocular pressure usually elevated

13. 12 year old boy presents with itching, blepharospasme, photophobia, blurred vision and copious mucoid discharge. Cobble stone is in upper tarsus. There’s family history of atopy. Which of the following is not correct :

a. Usually persist year around ion tropical climate.b. Involve Hypersensitivity reaction type II c. Horne trantas dotes are developed in limbal typed. Antihistamine is successful to treat mild casee. Steroid pulse tx is recommended for exacerbation condition

14. A women 27 yo, presents with severe progressive ocular pain and photophobia. She use contact lens during daily activity. From examination, dendritic epithelial lesion and grey white superficial non suppurative stromal infection is observed. Radial perineuritis was also found. The disease shown no response to variety topical antimicrobial/antiviral agent or steroid. Which of the following is not correct for the disease :

a. Diaminides is recommended for the treatmentb. Giemsa staining can be use to visualized the cause of the diseasec. The organism can be killed by freezing.d. Imidazoles is also drug choice of treatmente. In that patient, 6-12 month treatment is needed.

15. Seorang anak berumur 2 tahun dating dengan keluhan dari ibunya berupa : takut sinar; mata nrocos dan rewel/ Pada pemeriksaan ditemukan : Visus sukar evaluasi; CVI dan PCVI ringan ; photophobia dan lacrimasi. TIO digital ++ tanpa esotropia ataupun leukokoria. Pemeriksaan yg harus dilakukan pada penderita ini untuk menegakkan diagnosa adalah :

a. Aplanasi tonometri.b. Pemeriksaan dengan lempeng plasidoc. Sensibilitas kornead. Fluoresin teste. Visual field

16. Suspect diagnosa penderita ini :a. Retinoblastomab. Glaucoma congenital.c. Keratokonusd. Keratoglobuse. Staphiloma kornea

17. Penanganan penderita ini adalah :a. Enukleasib. Eviscerasic. Gonioskopid. Goniotomi.e. Iridektomi

Page 4: Contoh Soal Baru

18. One child of 10 yo suffered of vernal conjunctivitis for 5 years. He preferred to used maxitrol eyedrops because it can release his itching and redness. Since one year he feels difficulty while he rides his bicycle. The possible diagnosis of this patien t are :

1. open angle glaucoma2. close angle glaucoma ..3. drug induced glaucoma4. congenital glaucoma

19. The management of this patint (case number 18) are :1. stoped the maxitrol eye drops2. anti inflammation orally3. anti glaucoma drug A4. antibiotic eye drops.

20. 25 yo patient visits the hospital with blurred vision. Visual actuity ODS is 20/50. The patient also suffered from brochial asthma and often using corticosteroid nasal spray during asthma attack. Cataract of the both lenses is found during eye examination. What is the type of the cataract in this patiens :

a. Nuclear cataractb. PSC.c. Lemellar cataractd. Capsular cataracte. Cortical cataract

21. A patient come to clinic with complain of red and pain of the right eye. The eye underwent cataract surgery a week ago. From examination, retained cataract material is found in anterior chamber. Cells flare and keratic precipitate is also present. IOP is 35 mmhg. What is the definitive therapy for the patient :

a. Anti glaucoma medicationb. Lens material extraction.c. Steroid topicald. Steroid systemice. Non steroid anti inflammation

22. A patient visits the hospital with complain of abrupt onset of pain and redness of the left eye. The eye also blurred since 3 years ago. Eye examination show corneal edema and flare of anterior chamber. Lens is hyper mature. IOP is 40mmHg. The angle is open and precipitate of white flocular material is found at the angle. What is the most possible diagnose of the patient :

a. Lens particle glaucomab. Phacomorphic glaucomac. Phacolytic glaucoma.d. Glaucomfleeken

23. A patient is under going preparation for cataract surgery. The axial length is 23 mm, average of keratometry reading is 40D and a constant for the lens is 118. The proper IOL power for this patient if calculated with SRK I is :

a. 24.50D.b. 22.00Dc. 23.50Dd. 23.00D

Page 5: Contoh Soal Baru

e. 21.50D

24. After cataract surgery, ocular hypertension is found in a patient. Seidel test shows minor leakage of aquous at the wound site, although adequate and secure wound apposition is observed. What is the most proper option to manage this patient :

a. Give pilocarpine eye dropb. Hard contact lensc. Avoiding steroid.d. Re suture of the wounde. No management is needed

25. Datang seorang ibu, usia 70thn, dengan keluhan benjolan pada kelopak mata kiri atas bagian tengah. Pada pemeriksaan benjolan kemerahan dan melekat kuat pada jaringan dibawahnya berdiameter 7 mm. Untuk menegakkan diagnosa, perlu dilakukan tindakan sbb :

a. Excisional biopsyb. Incisional biopsy.c. Fine needle aspiration biopsyd. Wide excisic dengan vriscoupee. Wide exicsic dan rekonstruksi mata

26. Pada soal di atas ternyata suatu keganasan adnexa, yaitu suatu Sebaceous Carcinoma. Atas dasar itu direncanakan :

a. Wide excisi dengan insisi 3-5 mm dari batas undulasi tumorb. Wide excisi dengan insisi 1-3 mm dari batas undulasi tumor.c. Wide excisi dengan insisi 5-8 mm dari batas undulasi tumord. Wide excisi dengan insisi 8-10 mm dari batas undulasi tumore. Wide excisi dengan insisi 10-15 mm dari batas undulasi tumor

27. Penderita wanita usia 45 tahun datang dengan keluhan mata kiri menonjol secara perlahan sejak 1 tahun yl. Penonjolan mata kiri secara axial dan tidak ada rasa sakit. Tajam penglihatan pada mata kitri 6/60 dengan koreksi, sedangkan mata kanan 6/6. segmen anterior ODS tenang dan fundus kedua mata masih dalam batas normal, tapi terdapat RAPD + OS. Penderita mempunyai riwayat menggunakan kontrasepsi pil dan suntik selama 15 tahun. Pemeriksaan penunjang dalam pengelolaan penyakit untuk mengetahui lokasi dan perkiraan jenis tumor adalah :

a. Hemangioma konjungtivab. Sclerectasiac. Basalioma.d. Melanoma malignae. Haemorragic nevus

28. Pemeriksaan diatas mengarah suatu meningioma. Pengukuran hertel didapatkan proptosis ringan pada mata kiri. Tidak didapatkan corneal exposure. Pengobatan pada penderita ini adalah :

a. Eksenterasib. Double enukleasic. Orbitotomi lateral.d. Observasie. Pemberian steroid

29. Tatalaksana awal penderita pseudotumor adalah:a. Pemberian steroid dosis tunggal 1-2 mg / kgBB / hari selama 14 hari.

Page 6: Contoh Soal Baru

b. Biopsi insisic. Eksisi tumord. FNAB e. Radiasi

30. Wanita 20 tahun datang ke poiliklinik dengan keluhan kelopak mata atas kiri lebih turun. Keluhan ini dirasakan sejak penderita lahir dan tidak bertambah berat. Pandangan mata kanan dan kiri tidak terganggu. Pemeriksaan apa yang saudara lakukan utnuk menentukan jenis tindakan yang spesifik

a. Margin – reflex distance 1b. Margin – reflex distance 2c. Levator Function.d. Upper eyelid creasede. Vertical interpalpebral fissure height

31. Penyebab tersering ptosis kongenital pada kasus diatasa. Myogenic.b. Aponeuroticc. Neurogenicd. Mechanicale. Traumatic

32. Pada ptosis kongenital paling sering dilakukan tindakan operasi 1. Muller’s muscle resection2. Levator resection3. Fasanella – servat procedure4. Frontlis suspension surgery.

33. Komplikasi tersering operasi blefaroptosis :a. Wound Dehiscenceb. Tarsal eversion.c. Over correctiond. Under correctione. Scaring

34. Seorang anak perempuan berumur 8 tahun datang dengan keluhan kedua mata gatal dan merah. Dari hasil pemeriksaan didapatkan tajam penglihatan OD 5/6 dan OS 5/7.5, pada eversi kelopak mata didapatkan papil-papil yang besar dan injeksi konjungtiva. Diagnosa pasien ini adalah :

a. Blepharitisb. Uveitis anteriorc. Dacryostenosisd. Konjuntivitis vernal.e. Keratitis

35. Pasien diatas kontrol ke poliklinik mata setelah 1 athun kemudian sengan keluhan mata kabur sejak 1 minggu yl. Dari anamnesa diketahui bahwa keluhan mata merah dan gatal masih sering hilang timbul dan pasien tidak pernah kontrol. Pada pemeriksaan lampu celah yang didapatkan adalah :

1. Defek epitel berbentuk oval dengan kekeruhan di stromal pada kornea di bagian sentral

Page 7: Contoh Soal Baru

2. Punktata epitel erosi pada kornea superior3. Folikel pada limbus kornea4. Simblepharon konjungtivaB

36. Seorang bayi laki-laki umur 8 bulan pada pemeriksaan didapatkan epifora, fotofobia, blefasrospame, dan edema kornea. Diagnosa kemungkinan paling besar adalah :

a. Epidemic keratoconjunctivitisb. Uveitis anterior ( Juvenile Rheumatoid Arhritis)c. Glaukoma kongenital primer.d. Keratitis bakteriale. Obstruksi duktus nasolakrimal

37. Seorang bayi laki-laki umru 3 bulan dibawa orang tuanya dengan keluhan mata putih yang baru disadari orang tuanya. Dari hasil pemeriksaan funduskopi didapatkan adanya gambaran persistent hyperplasic primary vitreous (PHPV). Hasil temuan ini bisa berhubungan dengan :

1. Glaukoma2. Mikropthalmia3. Hipoplasia Fovea4. Ablasio retina E

38. Seorang anak perempuan berumur 2 tahun datang ke poliklinik dengan diantar kedua orang tuanya, dengan keluhan pada mata kanan menonjol, teradapat warna keputihan disertai mata merah dan anak sering menangis dan rewel. Dari hasil gambaran USG didapatkan echo high spike dangen kalsifikasi di dalamnya. Diagnosa yang paling mungkin :

a. Katarak kongenitalb. Glaukoma kongenitalc. Endoftalmitisd. Retinoblastoma.e. Panuveitis

39. Anak laki-laki 10 tahun datang dengan keluhan juling, tidak terus menerus dan hanya muncul jika penderita tidak konsentrasi dan melamun, dan juling bergantian. Dari hasil pemeriksaan diadaptkan eksotropia intermitten tipe basic dengan kontrol yang baik. Penatalaksanaan penderita ini berupa :

1. Reses otot rektus lateral2. Pemakaian lensa minus3. Resek otot rektus medial 4. Terapi push-up C

40. Tindakan operatif bedah strabismus dapat menimbulkan komplikasi :1. Perforasi sklera2. Diplopia3. Conjuctival inclusion cyst4. Dellen E

Page 8: Contoh Soal Baru

STRABISMUS 1. A 7 month old infant boy demonstrates the ocular deviation is esotropia. Each eye is able to

abduct normally. The child was born after a normal, full term pregnancy by a normal delivery. Considering the most likely diagnosis, the following statements is true : a. Vertical deviations are commonly associated b. Amblyopia isn’t commonly associatedc. Abduction movements often symmetricd. Adduction movements often symmetrice. Patients with this condition rarely require glasses during childhood

2. A mother complains that her 6 year old daughter has had increased wandering of her left eye over the past year. Your examination notes an intermittent exotropia of 35 prism diopters with fixation at distance and 5 prism diopters of exophoria with ixaton at near. Her uncorrected visual acuity is 20/20 OD and 20/20 OS. After discussion with her mother, you decide on surgical treatment. Which of the following tests would be least helpful in surgical management of this patient a. 30 minute occlusion test b. + 3.00 lens test c. + 5.00 lens test d. Worth four dot test e. Cycloplegic refraction

3. After a 6 mm bilateral lateral rectus recession for an intermitten exotropia of 25 prism diopter, the patient develops an esotropia of 40∆ with constant diplopia and inability to abduct OD beyond the midline. The most likely cause is : a. Muscle slippageb. Accomodation spasmc. Surgical increase in AC/A ratiod. Inadvertent trauma to right sixth nerve during surgery

4. A 4 year old child with horizontal nystagmus sees 20/50 whwn her head is held straight but can see 20/25 by adopting a 45 ° left head turn. The nystagmus is unchanged when fixing on a near target. Which surgical approach would be most likely to reduce her head turn ? a. Recess the right medial rectus and the left lateral rectus mucles

Resect the right lateral rectus and the left medial rectus muscles.b. Recess both medial rectus muscles and resect both left lateral rectus muclesc. Recess the left medial rectus and the right lateral rectus mucles

Resect the left lateral rectus and the right medial rectus muscles.d. Recess the left medial rectus muscles and resect the left lateral rectus muclese. Recess the right lateral rectus and the left medial rectus muscles.

5. A 5 year old has a visual acuity of 20/20 OD and 20/200 OS. Motility examination reveals a 6 Δ esotropia at both distance and near. The 4 Δ base out prism test over the right eye is most likely to show : a. No shift of the left eyeb. Slow shift of the left eye temporally, followed by a fast convergence movement nasallyc. A fast shift of the left eye temporally, followed by a slow convergence movement nasallyd. A fast temoral shift of the left eye without refixation movement

Page 9: Contoh Soal Baru

6. A 25 year old male accountant sustained a head injury as a result of an automobile accident 9 monhs previously. This patient’s chief complaint is double vision when reading. There is no deviation in primary gaze, but a 15 prism diopter esotropia is present in down gaze. There is a left hypertropia of 8 prism diopters in right gaze and a right hypertropia of 10 prism diopters in left gaze. Excyclotorsion of 6° is present in primary gaze, increasing to 15° in down gaze. Ocular versions demonstrate underaction of both superior oblique muscles. What is the best treatment option for this patien? a. Reading glasses with prismsb. Bilateral superior oblique tucksc. Bilateral inferior oblique weakening proceduresd. Advancement of the anterior fibers of both superior oblique musclese. Advancement of the anterior fibers of both inferior oblique muscles

7. A 44 old man sustained closed head trauma and now complains of object appearing tilted. The degree of tilting can be quantified by which of the following test?

a. Simultaneous prism-cover testb. Careful analysis of ductions and versions togetherc. Lateroversion reflex testd. Double Maddox rod test

8. Which of the following can not be used to test for ARC?a. Worth four dot testb. Major amblyoscope testc Titmus stereo testd. Cuppers monocular afterimage test

9. A 6 month old girl with esotropia in a healthy child is generally recommendeda. When the child is over 2 years old so that inferior oblique overaction and dissociated vertical deviation can be treated simultaneouslyb. When the child start to walkc. When amblyopia has been treated and deviation is of stable magnituded. When the child is 4 month old so as to maximize the chance for developing stereopsis

10. A 3 year old boy with a histry of congenital esotropia surgically corrected at age 12 months has visual acuity of 20/30 with both eyes open, but only 20/60 whwn each eye is tested monocularly using an occluder. The most likely reason for this discrepancy is a. Bilateral amblyopiab. Latent nystagmusc. Crowding phenomenond. Binocular summation

TUMOR1. A patiens 40 year-old with The malignant lesions most frequently affecting the eyelids are :

1. Basal cell carcinoma2. Squamous cell carsinoma3. Sebaceous cell carsinoma4. Melanoma

Page 10: Contoh Soal Baru

2. A 1 year-old presents with a round, well-demarcated mass at the supero temporal rim. The lession has been present since birth. The most lakely diagnosis is

a. Rhabdomyosarcomab. Neurofibromac. Dermoid cystd. Capillary hemangiomae. Metastatic Ewing sarcoma

3. A 65 year-old women presents with a progressively enlarging mass in the right inferior orbit. Distraction of the lower eyelid reveals a “Salmon patch” appearance to the fornix. The most lakely diagnosis is

a. Reactive lympoid hiperplasia b. Sebaceous carcinomac. Melanomad. Limfomae. Apocrine hidrosarcoma

4. Which of following signs is most likely to the present in patient with Graves ophthalmopathya. Exopthalmusb. External ophthalmoplegiac. eyelid retractiond. optic neuropathy

5. A 20 year-old man is struck over the right eye, and radiography show a fracture of the right orbital floor. Forced traction testing is equivocal because of poor cooperation. Four day after injury, 3 mm of the right exophthalmos is present , and movement of the eye are restricted in up gaze, down gaze, and horizontal gaze. Treatment at that time should be

a. Conjungtival incision throught the inferior fornix, with examination of the fracture b. caldwell-luc incision and packing of the maxillarry sinus

c. Skin incision over the inferior orbital rim and covering of the fracture defect with a plastic plate

d. Skin incision beneath the eyelashes and covering of the fracture defect with a plastic plate

e. None of the above

6. The most important determinant in selecting a corrective procedure for any type of ptosis is a. vertical height of the palpebrak tissueb. Age of the patientc. Amount of the levator functiond. Duration of the ptosise. Position of upper eyelid margin relative to the corneal limbus

7. A 70 year-old woman has 4 mm of right upper eyelid ptosis and 1 mm of left upper ayelid retaction. She has a heig eyelid crease in the right upper eyelid with normal levator function of both upper eyelids. Treament of the choice is

a. A moderated internal tyarsoconjungtival resection (fasanella-servat operation) on the right upper eyelidb. A moderate levator recession of the left upper eyelidc. A levator aponeurosis advancement on the right upper eyelidd. A posterior approach, standart mullerectomy on the right upper eyelid

Page 11: Contoh Soal Baru

e. A frontalis muscle suspension on the right upper eyelid using a silicone rod to allow postoperative adjustment

8. A 40 year-old woman presents with a progresively enlarging clear cystic mass along the eye lid diagnosis is

a. Epidermal inclusion cystb. Apocrine hidrocytomac. Siringomad. Tricholofolliculomae. Basal sel carsinoma

9. A 3 year-old girl was bitten by her pet dog. A 6-mm-wide block of upper eye-lid margin is hanging by a thread of tissue. This block of tissue left a defect in the upper eyelid approximately the same size as the tissue it self. The best do would be to

a. send the block of tissue to pathology and repair the defect by approximating the two margin

b. send the block of tissue to pathology and repair the defect by making a lateral canthotomyso that the skin adges ccan be approximata

c. Repair the eyelid by sewing the block of tissue into its normal anatomic positiond. send the tissue to pathology and repair the defect by a tranfer of the tissue from the lower

eyelide. Keep the tissue under refrigeration for later use if necessary and close the defect by

approximation of the wuond edges and lateral canthotomy

10. A 74 year-old woman present with a 2-year history of the painless, progresively enlarging mass in the central aspect of the upper eyelid. This has resulted in distortion of the eyelid margin and loss of eyelashes. The most likely diagnosis is

a. Sebaceous gland carcinomab. Squamous cell carcinomac. Amelanotic melanomad. Basal cell carcinomae. dermal nevus

11. Six hour after blepharoplasty, the patient complains of sudden pain near the right eye. The dressing are removed and the right eyelids are tense and ecchymotic. The first step would be to

a. Open the wound to release a possible retrobulber hemorrhageb. Consider the possibility of cavernosus sinus thrombosis and corneal sensationc. measure visual acuity and check pupillary respond. Begin treatment with ice packse. Begin treatment with warm compresses

12. A 75 year-old woman complains of tearing and discharge. Irrigation of lower canalliculus produces mucopurulent reflux. All the following are true except ;

a. Jones testing will not reveal dye in the noseb. There is the probably a common canalicular blockc. The condition is not likely to resolve with a course of antibioticsd. The most likely diagnosis is a lacrimal duct obsructione. The treatmen is dacryocystorhinostomy

Page 12: Contoh Soal Baru

13. A 14 year-old child has had tearing and discharge from the right eye since birth. Which the following statements is true?

a. dye disappearance testing is likely to show no asymetryb. This condition is likely to resolve spontaneuslyc. The appropriate treatment is nasolacrimal duct probingd. Punctal abnormalities are likely to be the causee. jones I and jones II test are necessary to make the diagnosis.

14. The management of rhabdomyosarcoma of the orbit usually involvesa. Lumbar puncture to rule out central nervuos system metastasisb. Exentration of the orbitc. Enucleation and orbital radiationd. Systemik chemotherapy and orbital radiatione. Radical neck disection if cervical limph nodes are involved

15. Congenital coloboma is frequently associated with :1. Facial clefts2. Eyelid margin inversion3. Lacrimal deformities4. Eyelid retraction dysgenesis

16. Acquired eyelid Disorder associated by :1. Chalazion2. Hordeolum3. Eyelid edema4. Flappy eyelid Syndrome

17. In patiens with a facial nerve paralysis, all of the following characteristics may be present except :

a. eyebrow ptosiswb. Blepharopthosisc. Lower eyelid ectropiond. Epiphorae. Ocular exposure symtom

18. A patient with Telecanthus and epicanthus in inversus are correted by :1.Y plasties2. Z plasties3. V plasties4. Y-V plasties

19. Severe congenital Ectropion may give rise to :1. Chronic epiphora2. Blepharophimosis3. Exposure Keratitis4. Ichthyosis

LENS & CATARACT

1. A patient with irregular astigmatism. The doctor will do corneal topography. The accuracy of corneal topography may be affected by various potential problems. Which one is correct:

Page 13: Contoh Soal Baru

1. Tear film effects2. Distortion3. Sensitivity to focus errors4. Decrease accuracy of corneal power simulation measurements after refractive surgical

procedure.

2. A 30 yo man will get corneal refractive surgery. Corneal refractive procedures can be classified as lamellar keratotomy, keratectomy, collagen shrinkage, or penetrating keratoplasty. These procedure can alter the corneal biomechanics in several ways:1. Incisional effect 2. Tissue addition or subtraction.3. Alloplastic material addition4. collagen shrinkage.

3. A patient with +6.0 D want a refractive surgery. Which one of type refractive surgery can you do for that patient:1. LASIK2. Intracorneal lens or ring3. Epikeratoplasty4. Keratotomy

4. A patient with astigmatism 5. 0 D. For this astigmatism case which one of this type is correct1. Keratotomy radial2. Collagen shrinkage3. Limbal relaxing incision4. Penetrating keratoplasty

5. The patient after RK can have many complication. Potentially blinding complications occur rarely after RK. These include:

1. Perforation of the cornea2. Bacterialkeratitis3. Traumatic rupture of the globe occurring through a keratotomy incision4. Vascularization of stromal scars.

6. In hexagonal keratotomy can result corneal ectasia.What is the visual problem can we find after procedure that induce corneal ectasia:1. Glarre2. Fluctuating vision3. Irregular astigmatism4. Polyopia

7. A Patient with refractive surgery procedure. In this procedure aplus power lens is placed intra stromally to increase the curvature of the anterior cornea for the correction of hyperopia. The lens can be prepared either from donor cornea ( homoplastic or alloplastic). What kind of that procedure:

a. Epikeratoplastyb. Intra stromal cornealring segment.c. Keratophakiad. Orthokeratology

Page 14: Contoh Soal Baru

8. One of many refractive surgery procedure that eliminate the complexity of the lamellar dissection and intra operative lathing of early keratomileusis procedure in which a corneal cap was dissected from the eye,shaped on a cryolathe and then repositionded with suture.What kind of this procedure:

a. Keratophakiab. Epikeratoplastyc. Orthokeratologyd. Intrastromal corneal ring segment

9. Epikeratoplasty was later expanded to include the treatment of hyperopia and myopia. The following is recommendated by FDA in 1998 for selection the patient:1. A visually disabled patient with keratoconus whois contact lenstolerant.2. A visually disabled monocular aphake for whom secondary IOL insertion is

contraindicated3. A visually disabled binocular aphake for whom secondary IOL insertion is

contraindicated4. A patch graft used for tectonic purposes.

10. In a refractive surgery procedure a circular rings PMMA are placed in the midperipheral corneal stroma in a lamellar channel. What is the procedure:

a. Keratophakiab. Intrastromal corneal ring segment.c. Keratophakiad. Orthokeratology

11. Intacs are approved by the FDA for low levels of myopia ( -1.0 D – 3.0D spherical equivalent) but cannot correct astigmatism. Additional selection criteriafor patient :1. With an age of 21 year or older2. With documented stability of refraction at least 12 month prior to the preoperative

examination.3. With 1.0 D of astigmatism or less4. In eye with a corneal keratotomy steeper than 46 D or flatter than 40 D.

12. For the myopia or hyperopia patient can get Intacs. Intacs are not recommended for this following:1. In patient with a low-light pupil diameter of 7.0 mm or larger.2. In patient with systemic disease likely to affect wound healing.3. In patient with a history of ophthalmic herpes simpleks or herpes zoster.4. with 1.0 D of astigmatism or less.

13. A post Intacs patient can get complication. The following is complication of Intacs:1. Anterior chamber perforation2. Microbialkeratitis3. Implant expulsion4. shallow ring segment placement

14. A patient with corneal refractive therapy refers to the overnight use of gas – permeable contact lenses to temporarily reduce myopia (-0.50 to – 6.00 D of sphere with up to 1.75 D of astigmatism).This methode is:

Page 15: Contoh Soal Baru

a. Orthokeratologyb. Keratophakiac. Epikeratoplastyd. Intrastromal corneal Ring segment.

15. Post operative dry eye is more common with LASIK than with PRK. How toperform preoperative evaluation:1. Tear meniscus2. Rose Bengal staining3. Schirmer testing4. History

16. Post LASIK patient with complication come to you. He is in stage 1 diffuse lamellar keratitis. What can you find:

a. Peripheral faint white blood cells, granular appearanceb. Central scattered white blood cells, granular appearancec. Central dense white blood cells in visual axis.d. Permanent scarringor stromal melting

17. A patient post LASIK with complication had flap melting, severe irregular astigmatism,corneal scarring and we should to lift that flap. What kind of the complication:

a. Sterile interface inflammationb. Infectious keratitisc. Epithelial ingrowthd. Interface debris

18. The doctor will do clear lens extraction. The following condition is suitable for this:1. Cornea too thin2. Cornea too flat3. Cornea too steep4. Refractive error exceeds the limit of excimer laser treatment.

19. LASIK and PRK are both commonly use to treat which refractive range:a. -14 to + 8 Db. -8 to + 4 Dc. -6 to +2 Dd. -5 to + 5 D

20. Which one of the following condition is an absolute contraindication to refractive surgery:a. dry eyeb. glaucomac. scleral bucklingd. Herpes viruse. Amblyopia with vision < 20 / 200.

Page 16: Contoh Soal Baru

CLINICAL OPTIC

1. A patient with low vision wearning +6.25+4.75 x 90 OD and +5.00+3.50 x 90 OS has an overrefraction (see the figure) of-1.00+1.25x180 OD and +0.50+1.00 x 90 OS at a distance of 1 m from a wall chart. The resultant distance refractive correction is

a. +7.50+3.50 x 90 OD, +6.50+4.50 x 90 OSb. +6.50+3.50 x 90 OD, +5.50+4.50 x 90 OSc. +5.25+6.00 x 90 OD, +5.50+4.50 x 90 OSd. +5.50+3.50 x 90 OD, +4.50+4.50 x 90 OS

2. At a distance of 1 m, the patient in Question OR1 reads the 6/9 line OD and the 6/6 line OS. The patient requests that you fill out a disability form.You record the acuity as

a. 20/30 OD, 20/20 OSb. 20/180 OD, 20/120 OSc. 20/100 OD, 20/80 OSd. 20/300 OD, 20/200 OS

3. Some patients with low vision have better reading function binocularly, wherears others read better monocularly. Which of the following is least accurate?

a. When both eyes have substantially asymmetric macular function (eg, OD 20/60, OS 20/400), retinal rivalry may limit binocularity at near, and thus may affect print locailzation, cause text jump, and worsen visual blur.

b. Binocular reading should be explored when the visual acuity of each eye is within two lines of each other.

c. Worth four-dot testing at near is helpful in assessing peripheral fusion and binocular low-vision potential

d. Asymmetric eccentric fixation enhances binocularity by providing similar extrafoveal acuity in each eye, regardless of retinal correspondence.

4. A 26-year old man had a cataract extraction with a scleral-sutured IOL 2 years ago, after he developed a traumatic cataract with marked zonular dehiscence. Six months ago, his eye was quiet and his best-corrected visual acuity was 20/30 with -1.25 sphere. Yesterday he was hit in the pseudophakic eye. Today he complains of bulrred vision, and a slit-lamp examination shows that lens is rotated about its vertical axis.Assuming the lens is not displaced anteriorly or posteriorly, you would expect him to be.

a. More myopic,with with-the-rule astigmatismb. Less myopic, with with-the-rule astigmatismc. More myopic, with against-the-rule astigmatismd. Less myopic, with against-the-rule astigmatism

5. In prescribing telescopic devices for low vision, you must consider the optical design of the telescopic system and the tasks to be addressed.Which of the following is true ?

a. The gallean telescope has a plus objctive and a minus ocular lens and produces a real and inverted image.

Page 17: Contoh Soal Baru

b. Positioning the telescope so that the exit pipil of the telescope coincides with the entrance pupil of the patient’s eye ensures the largestfield of view and maximum light transmission.

c. Aligning the telescope’s optical axis with the patient’s optical axis minimizes distortion. Even when the patient has an accentric visual axis of an extrafoveal preferred retinal locus.

d. Alow-power minue lens used as a “reading cap” over the objective is positioned inside the telescope’s exit pupil (between the optical center and the focal point) to reduce individual accommodative effort and permit a closer near focus.

6. Contrast sensitivity loss in the low-vision patient may adversely affect the performance of specific tasks. Which of the following statements is least accurate?

a. High-freguency contrast sensitivity loss particularly aggravates near point tasks and reading performance.

b. A 3x magnifier with supplemental halogen illumination can rival a. 5x magnifier using only ambient lighting.

c. Telemicroscopes (eg,loupes) are often helpful when low vision is accompanied by impaired contrast sensitivity.

d. Travel-related tasks and mobility issues are most affected by low-frequency contrast sensitivity loss.

7. You examine a healthy 6-month –old child with uncertain visual acuity but definitely showing a reduced visual interest. The clinical examination is unrevealing, and further testing is scheduled. Which of the following is most accurate regarding the child’s visual impairment and its educational implications?

a. The child should be referred immediately to the district school system so special education assessment for visual impairment can begin.

b. The absence of nistagmus now and over the next 2years is an indication that special education vision sevices are probably not necessary now and will not be necessary in the future.

c. Federal law mandates that the visually impaired child, beginning at age 3, be identified to the special education system.

d. The physician should wait to refer a child with a possible visual impairment to the special education system until the diagnosis is established or legal blindness is confirmed.

8. Low-vision rehabilitation is distinguished in ICD-9 coding by the level of visual impairment. Which of the following is not true regarding the 369.xx codes?

a. Moderate low vision is defined as 20/70 to 20/200 acuity.b. Severe low vision is defined as 20/200 to 20/400 acuity.c. Profound low vision is defined as less than 20/400 acuityd. A specific visual field restriction, although found in the definition of legal blindness,

is not a consideration in ICD-9 coding of the visual impairment.

9. An air traffic controller with 4 D of myopia and 1 D of accommodative amplitude wants single-vision glasses for viewing a video screen 80 cm in front of her. If she uses half of her accommodative reserve, what power lenses should she be given?

a. -3.00 sphb. -2.75 sphc. -3.25 sphd. -3.50 sph

Page 18: Contoh Soal Baru

10. A new patient wishes to be fitted with cantact lenses for an upcoming ski trip. He plans to wear the contact lenses only for recreational activities. His refractive correction is -6.00 + 0.25 x 090 OU. Which type of lenses should you recommend?

a. PMMA lensesb. Spherical soft hydrogel lensesc. Rigid gas-permeable (RGP) lensesd. Toric soft lenses

11. While performing retinoscopic on a 3-yeard-old, the retinoscopic reflex is neutralized by -3.00 sph OD and +3.50 sph OS. Assuming a working distance of 67 cm, the best prescription to give is?

a. -3.00 sph OD, +3.50 sph OSb. -4.50 sph OD, plano OS c. -4.50 sph OD, +2.00 spg OSd. -3.00 sph OD, plano OS

12. An asymptomatic 6-year-old has uncorrected snellen acuity of 20/50 OD and 20/40 OS. Ductions and versions are full, and the child is orthophoric at near and distance. Cycloplegic refraction of OD +1.00 + 4.50 x 045 and OS + 0.50 + 4.00 x135 yields no improvement in acuity. The best course of treatments is.

a. Not to precsibe glasses there is no change in acuity with correction.b. To prescribe + 0.75 + 4.50 x 045 OD, and + 0.25 + 4.00 x 135 OSc. To prescribe + 0.75 +2.25 x 045 OD, and + 0.25+2.00 x 135 OS because the child

might not tolerate the full amount of oblique cylinder.d. To prescribe the spherical equivalent correction because there is no change in acuity

with correction.

13. The boy in the figure on page 320 was hit in the fece with a soccer ball this morning and is complaining of binocular vertical double vision. At this last visit, he had normal vision with correction and no strabismus. The plastic surgeon wants to repair a small left orbital floor fracture that was demonstrated on a CT scan of the orbits. Before operating, the plastic sur-in each eye, and eye movements are full. If his diplopia is due solely to spectacle misalignment, you would expect that?

a. The effective bese-up prism OS is causing a left hypertropia on cover testing.b. The effective base-down prism OS is causing a right hypertropia on cover testingc. The effective base-down prism OS is causing a left hypertropia on cover testingd. The effective base-up prism OS is causing a right hypertropia on cover testing.

14. A 51-year-old electrical engineer has a refraction of -250 + 0.25 x 161 OD and -2.25 + 0.75 x 016 OS; his accommodative amplitude is 1 D. He has bifocals but complains of trouble with his intermediate vision. You write a prescription for trifocals, with an intermediate of +1.50 and a near of +3.00. He returns a week later complaining of “gaps” in his clear vision between

a. 67 and 100 cm, 25 and 40 cmb. Infinity and 67 cm, 40 and 33 cmc. Infinity and 40 cm, 33 and 25 cmd. 67 and 100, 40 cm and 33 cm

15. A previously emmetropic and orthotropic patient is given the following glasses after bilateral cataract surgery:-0.50-2.00 x 179 OD, +0.50+1.00 x 180 OS, with + 2.50 add OU. If the

Page 19: Contoh Soal Baru

patient reads I cm below the optical centers of his lenses, the anticipated deviation in the reading position is

a. a 4∆ right hypotropia b. a 1∆ right hypotropiac. a 4∆ right hypertropiad. a 1∆ right hypertropia

16. A 62 year-old retired real estate salesman consults you because of intermittent “blurring” and binocular diplopia while reading and driving. He often shuts his right eye to see better. He is on no medications and past medical and surgical history are unremarkable. After further questioning, the patient reports macropsia OD. Which of the following statements about spectacle correction is most appropriate?

a. Iseikonic spectacles are indicated to treat his macropsiab. One prism diopter base-out prism OU (total 2∆) should be perscribed to help his

esotropia.c. Polycarbonate lenses should be prescribed, even if refraction does not improve his

acuity.d. Ultraviolet-filtering lenses should be recommended to prevent further progression of

his age-related macular degeneration.

INFEKSI IMUNOLOGI

1. A 55 y.o patient presents in your office with complaints of decreased vision. On examination, you detect severe anterior basement membrane dystrophy and a cataract. Which one of the following tests would be most helpful to you to determine whether the main cause of decrease vision is the corneal pathology or the cataract?

a. Potential acuity meterb. Corneal topographyc. Contact lens overrefractiond. Glare testing

2. A healthy 60 y.o man present with a 2 day history a painful rash on the right side of his forehead extending down to the eyelids. A vesicular skin lesion is also seen near the tip of his nose. Which of the folowing therapies would be most appropriate?

a. Topical trifluridine 1% drops 8 times per day for 14 daysb. Oral famciclovir 500 mg 2 times per day for 10 daysc. Oral valaciclovir 1000 mg 3 times per day for 10 daysd. Oral acyclovir 800 mg 3 times per day for 10 days

3. A 41 y.o Japanese man with a remote history of blunt trauma in one eye but good vision and no history of ocular surgery presents with decrease vision and severe pain in both eyes. He has bilateral uveitis, alopecia, vitiligo, and recent cerebrovascular accident. There is an exudative retinal detachment in one eye. Which of the following diagnoses is the most likely?

a. Sarcoidosisb. Sympathetic ophthalmiac. Behcet syndromed. VKH syndrome

4. A 67 y.o white female present with mild uveitis with mild vitritis and subretinal infiltrates. The condition has been minimally responsive to topical corticosteroid treatment. She has

Page 20: Contoh Soal Baru

recently experienced weakness and confusion. Which of the following tests would be the most important to obtain at this time?

a. Gallium scanb. CT-scan or MRI of the headc. PPD and chest x-raye. Westergren sedimentation rate and C-reactive protein

5. A patient with a previous mitomycin C trabeculectomy present with a severe bleb related endophthalmitis . The VA previously was 20/20 and is now hand movements. Which of the following is not correct?

a. The visual prognosis is poorb. The organism in a bleb –related endophthalmitis is more likely to be H influenzae or

a streptococcus species than it is in a post cataract surgery endophthalmitisc. Because the VA is better than light perception, a vitreous tap for cultures and

injection of antibiotics should be performedd. Endophthalmitis may present months or years after glaucoma filtering surgery

6. A 35 y.o Japanese women present with slate gray pigmentation of the episclera of the right eye with association pigmentation of the periocular skin. Which of the following statement applies to her?

a. Her lifetime risk of uveal melanoma is about 1 in 400, significantly greater than the risk of the general population

b. Malignant melanoma can develop in the skin, uvea, or orbit but not in the conjunctiva

c. The risk of malignant transformation is the same in all patients regardless of the their complexion

d. This condition is always unilateral

7. A 33 yoman was involved in a motor vehicle accident that resulted in a broken windshield. Slit-lamp examination reveals multiple glass foreign bodies embedded at various levels of the stoma. All wounds were seidel negative. Which of the following is the most appropriate treatment strategy?

a. The patient should be taken to the operating room and all superficial and deep fragments should be removed immediately

b. Ointment should be place in the eyes since superficial and deep fragment will eventually be extruded with the time

c. Eyes should be patched with the follow up the next dayd. All exposed glass fragment should be removed and deeper fragments left in place

8. Which of the following can be related to spontaneous hyphema?a. Herpetic diseaseb. Retinoblatomac. Juvenile xanthogranulomad. All of the above

9. Currenty the most prevalent refractive surgery is :a. Myopic LASIKb. Hyperopic LASIKc. Cataract removal with IOL implantation

Page 21: Contoh Soal Baru

d. LASEK

10. All of the following organisms can invade an intact corneal epithelium except : a. Pseudomonas aeruginosab. N meningitidisc. C diphtheriaed. Shigella

11. A hospitalized patient on IV antibiotics for pneumonia develops floaters and blurred vision in one eye. Funduscopic evaluation reveals multiple yellow white choroidal lesion with indistinct margin and vitreous fluff balls. Which of the following presents the most appropriate management?

a. Inquire whether the patient has had recent exposure to cats or uncooked meatb. Culture the patient’s blood and catheter tips on multiple media and begin IV

antifungal medicationc. Supplement the IV antibiotics with topical and subconjunctival antibiotics to

increase ocular penetrationd. Prescribe systemic corticosteroid to suppress the vitritis

12. A 64 y.o woman who had undergone cataract surgery 6 months earlier develops an anterior chamber reaction and low grade anterior vitreous inflammation following Nd:YAG laser capsulotomy of a white plaquelike opacity of the posterior capsule. The uveitis initially respon to topical steroids but recurs as the steroids are withdrawn.The most likely diagnosis is

a. Phacoantigenic endophthalmitisb. Sympathetic ophthalmiac. Uveitis-glaucoma-hyphema syndromed. Propionibacterium acnes endophthalmitis

13. Which of the following patients would benefit most from steroid therapy?a. A 9 y.o girl with JRA,moderate anterior chamber flare, but no cellb. A 22 y.o woman with pars planitis,anterior vitreal cells,but no CMEc. A 44 y.o woman with Fuch heterochromic iridocyclitis, fine KP, and anterior

chamber celld. A 50 y.o man with herpes simplex keratouveitis, disciform corneal edema, keratic

precipitate, and chamber flare cell

14. A 64 y.o man developsbilateral decreased vision and floaters. Examination reveals clumps vitreous cells; subretinal infiltrates; and a white quiet eye.The vitritis is not responsive to steroid. Themost likely associated finding would be

a. Tumor cells in the CSFb. Positive VDRL serologyc. Hilar Lymphadenopathy on chest x-rayd. Pneumocystis carini in sputum specimens

15. Peripheral retinal granuloma can be found in all of the following except :a. Ascariasisb. Toxoplasmosisc. Sarcoidosisd. Syphilis

Page 22: Contoh Soal Baru

16. Diagnostic criteria supporting a diagnosis of Behcet syndrome include all of the following except :

a. Genital ulcersb. Oral ulcersc. Keratoderma blennorrhagicumd. Erythema nodosum

17. A 9 y.o boy with a history of atophy present with a seasonally recurrent bilateral conjunctivitis and complains of blurred vision for 1 week. Giant papillae are seen upon lid eversion. All of the following could also seen on the slit lamp exam except

a. Vascullar pannus and punctate epithelial erosion involving the superior corneab. An oval epithelial ulceration with underlying stromal opacification in the central

corneac. Limbal folliclesd. Conjunctival symblephara

18. A 30 y.o woman with chronic irritation in both eyes is found to have mild papillary conjunctivitis, punctuate staining of the superior cornea and conjunctiva of both eyes and filaments attached to the upper limbus of one eye. Bulbar conjunctival biopsy of the involved area showed keratinization, acanthosis, intracellular glycogen granules. Which of the following diagnostic tests would be most appropriate?

a. Thyroid functions testingb. Serum RFc. HLA typingd. Histamine level of the tear film

19. Steven Johnson Syndrome a. Is also referred to as erythema multiforme minorb. Is generally a disease of elderly patientsc. Is commonly associated with drug hypersensitivityd. Rarely presents as an acute febrile episode

20. A 59 y.o woman presents with the recent of a flat, darkly pigmented macule of the left lower palpebral conjunctiva. Of the following conditions, which would be most likely?

a. Blue nevusb. Primary acquired melanosisc. Benign acquired melanosisd. Lentigo senilis

CLINICAL OPTIC

1. A patient comes for refractive surgery with keratometry readings of 43,0 D/ 42,0 D and a manifest refraction of -9,5 D. If LASIX were performed,you would expect the postoperative average keratometry reading to be : (102)

a. 36,8 D b. 36,3 D c. 37,3 D d. 34,0 D

2. The clinician should be familiar with levels ofillumination frequently recommended by engineers. Recommended illumination levels are (in foot candles) : (16-17)

1. Office : 150 2. Kitchen :70 3. Projector : 104. Operating table : 250

Page 23: Contoh Soal Baru

3. Retinoscopy is usually performed using the plano mirror setting. Characteristics of the retinoscopy reflex are : (128)

1. speed 2. width 3. brilliance 4. colour

4. A six years old boys with hyperopia unless there is esodeviation or evidence of reduced vision,our management is: (146-147)

a. It is not necessary to correct hyperopiab. full correction according to cycloplegic retinoscopyc Minimum correction according to cycloplegic retinoscopyd. Correction is waiting for until 8 years olde. Correction without cycloplegic

5. Magnification of approximately 20-30%, altered depth perception resulting from the magnification, pincushion distortion, ring scotoma generated by prismatic effects at the edge of the lens, & “jack-in-the-box” are some problems of correcting : (164)

a. contact lens b. RGP c. Prebylasix d. Aphakia e high myopia

6. Monocular Diplopia is frequent and confusing complaint in general ophthalmic practice, causes by : (170-171)

1. Irregular astigmatism 2.decentered contac lens3. Double reflection inspectacle lenses 4. Post strabismus surgery

7. Key optical considerations resulting from contact lens use are : (177)1. Field of vision 2. Image size 3. Accomodation 4.tear lens

8. Conditions and circumstances where bandage contact lenses might be useful include : (202) 1. Bells palsy 2.Reccurent erotions 3. Bullous keratopathy 4.Keratitis

9. Sometimes an IOL maybe inserted in an eye that already has an IOL, usually to correct a postoperative refractive error. In this case, theoretically, there is no change in any of the optical parameters of the eye,so calculation of the needed second IOL power is straightforward geometrical optics : (224-229)

a.Multifocal IOL b.Piggyback IOL c. Standard IOLd. Diffractive multifocal IOL e.Accomodating IOL

10. A small change of vertex distance produces considerable blurring of vision,especially for patients with : (144)

1. An isometropia 2. High myopia 3. Silicon IOL 4. Aphakia

Page 24: Contoh Soal Baru

SOAL – SOAL BOARD UNAIR JAN 08

1. A 20 years old woman came with redness on her both eyes, she also suffer fom rhinitis allergic and asthma. Which one is the most signified for this symptom:

a.Foreign body sensationb.Itchingc.Sandy feelingd.Deep paine.Lid pain

Answer : B (AAO 2006-2007 section 8 Externa disease and cornea : pg. 208)

2. Patiens came with red nodular mass, location surrounding the eye lashes, she complains of tenderness and painful. The diagnosis is :

a. Internal hordeolab.Styesc.Chalaziond.Periorbital celulitise.Meibomian glan dysfunction

Answer : B (AAO 2006-2007 section 8 Externa disease and cornea : pg. 156)

3. The management of the disease above is :a.Cultureb.Topical antibioticc.Systemic antibioticd.Warm compresse.All above is true

Answer : D (AAO 2006-2007 section 8 Externa disease and cornea : pg 157)

4. A 6 years old girl present with diffuse, erythematous & edema of the eye lids. Lids are tenderness to touch and swollen, visual acuity and ocular motility are normal. The most common organism can cause this problem is:

a.Streptococcus epidermidisb.Staphylococcus aureusc.Klebsiella sp.d.Neiseria meningitidise.Pseudomonas aeruginosa

Answer : B (AAO 2006-2007 section 7 orbit eyelid, lacrimal system : 41-42)

5. A man, 35 years old present with serous discharge and red eye. Which of the folowing statement aplies to him:1. pattern of redness tends to be concentrated in the palpebra are rather than near the cornea2. palpable lymph node auricular

3. discharge swab revealed lymphosite dominant4. folicular conjugtivitis

Answer :E (AAO 2006-2007 section 8 Externa disease and cornea : pg 24-25)

6. A 65 years old lady who consumes many drugs for her diseases complained burning, gritty sensation, reflex tearing and mild redness.

Which of these drugs can influence her symptoms?1. systemic antihistamine2. diuretic3. antidepresant4. non steroid inflamatory drug

Answer :B (AAO 2006-2007 section 8 Externa disease and cornea: pg 81)

7. We performed several examination for her, and revealed :1. tear meniscus < 0,1 mm2. break up time more rapid3. schirmer test < 5 mm4. fine and granular epithelial keratopathy was shown by Rose Bengal

Answer : E (AAO 2006-2007 section 8 Externa disease and cornea: pg 73)

Page 25: Contoh Soal Baru

8. A Female model, 17 years old with cosmetic contact lens wear present with redness, tearing , pain, photophobia and flourescein test positive at 3 and 9 o’clock in the cornea. The management were:a. prescription of topical anesthesiab. cyclopentolate 1% topicalc. anti inflamation eye drop 3 tmes dailyd. antibiotic ointment

Answer : C (AAO 2006-2007 section 8 Externa disease and cornea: pg 406-407)

9. Corneal dystrophy of Bowman’s layer isa. Previously known as Reiss Buckler dystrophy and Thiel Behnke dystrophy b. Progresive autosomal dominanc. Thiel Behnke dystrophy demonstrated curly fibers on electron microscopy and is linked tochromosom 10q24d. Appears at third decade of life

Answer : A (AAO 2006-2007 section 8 Externa disease and cornea : 315)

10. Macular dystrophy is a. autosomal recessive inheritanceb. amyloid deposit concentrated in the anterior stromac. gene identified on chromosom 16 q22d. affected the central corneal and do not involve cornea endotheliumAnswer : B (AAO 2006-2007 section 8 Externa disease and cornea : 310)

11. Which one of these component imunology in healthy conjungtiva is absent:a. neutrophilb. lymphositec. eosinophyld. macrophagee. plasma cell

Answer C (AAO 2006-2007 section 8 Externa disease and cornea : 195)

12. A man of 20 years old, came to the hospital with complain right red eye, tearing, eyelids edema with normal visual acuity . A few red eye disorders can threaten vision,except:

a.Corneal infectionb.Scleritisc.Iritis d.Blepharoconjungtivitise.Orbital celulitis

Answer : D (AAO: Eye care skills on CD ROM 2001 slide 1.)

13. The cornea’s imuno previlege is due to multitude of factors, including :1. absence of blood vessels2. absence of lymphatics 3. expression of neuropeptide such as alpha-Melanine Stimulating Hormone (alpha MSH) in the cornea4. absence of expression Transforming Growth Factor

Answer A (AAO 2006-2007 section 8 Externa disease and cornea : 196)

14. Graft versus Host disease is a relative common complication of allogeinic bone marrow transplantation. In this condition, the grafted cell attack the patient’s tissues including the eyes. Which tissues the eye can affected?

a. Palpebrab. Conjungtivac. Uvead. Lacrimal gland

Answer C (AAO 2006-2007 section 8 Externa disease and cornea : 223-224)15. Some disorders give symptom of photophobia that indicates problems arising from anterior segment . Which of the

following condition can cause photophobia?:a. corneal abrasion

Page 26: Contoh Soal Baru

b. iritisc. acute glaucomad. corneal edema

Answer : A (AAO: Eye care skills on CD ROM 2001 slide 5)

16. Interstitial keratitis is nonsupurative inflamation of the corneal stroma that features cellular infiltration and usually vascularization without primary involvement of the epithelium or endothelium. The most common cause to be linked with interstitial keratitis is :a. M tuberculosab. Rubeola (measles)c. Clamydia trachomatisd. Onchocerciasise. Syphilis

Answer ; E (AAO 2006-2007 section 8 Externa disease and cornea : 226)

17. A 51 years old lady present with a chronic, progressive, painful, idiopathic ulceration in peripheral corneal and epithelium. The ulcer starts at periphery of the cornea and spreads circumferentially and then centripetally, with leading undermined edge of de-epithelized tissue.

This lady suffered from :a. idiopathic peripheral ulcerative keratitisb. Superior Limbus Keratitisc. Ulcus moorend . Ulcus marginalise. Keratoconjungtivitis Sicca

Answer : C ( AAO 2006-2007 section 8 Externa disease and cornea :232)

18. Which one following examination is not present in this disease:a. Deficiency of Supressor T cellb. Increased level of Ig Ac. Increased level of Ig Gd. Increased concentration of plasma cells and lymphocytes in the conjungtiva adjacent to the ulcerated arease. Tissue-fixed imunoglobulins and complements in the conjungtival epithelium and peripheral cornea

Answer : C (AAO 2006-2007 section 8 Externa disease and cornea : 232)

19. The conjungtival biopsy can be helpful in evaluating chronic conjungtivitis and unusual cular surface diseases. Which one of the following condition unnecessary to do conjungtival biopsy :a. Superior limbic Keratitisb. Pterygiumc. Conjungtival lymphoid tumorsd. Graft versus Host diseasee. Cicatrical pemphigoid

Answer : B (AAO 2006-2007 section 8 Externa disease and cornea: 427)

20. Seorang pemuda berusia 18 tahun datang dengan riwayat kecelakaan lalu lintas 1 jam sebelum ke Rawat Darurat. Dari pemeriksaan didapatkan laserasi pada kornea mata kanan. Hasil pemeriksaan dibawah ini dapat memastikan adanya perforasi bola mata yaitua. Bilik mata depan dangkalb. Kemosis konjungtivac. Hipotonid . Seidel tes positife. Perdarahan subkonjungtiva

Answer : D (AAO 2006-2007 section 8 Externa disease and cornea: 409)

21. The principal photodisruption laser used in clinical ophthalmology is :a. Cardondioxideb. Carbonc. Nd: YAG laserd. Argone. Hollonium

Page 27: Contoh Soal Baru

Jawab : C

22. An Object positioned inside the focal point of a plus spherical lens will produce image thata. Minified, inverted, realb. Minified, upright, realc. Magnified, inverted,virtuald. Magnified,upright,virtuale. Magnified,upright, real

Jawab : D

23. Starting point of reading add for low vision patient with 6/24 best corrected vision is ..a. +3.00 Db. +3.50 Dc. +4.00 Dd. +5.00 De. +6.00 D

Jawab : C

24. A modified A-constan of an IOL for patient with axial length of 23,5 cm by using SRK II formula isa. A+3b. A+2c. A+1d. A-Ae. A-0,5

Jawab : D

25. Equivalent of two spherocylindrical lens +1.00 -2.50 x90 and +1.00 +2.00 x180a. +2.00 -0.50 x 90b. -0.50 -4.50 x 180c. +1.50 +0.50 x 180d. +1.00 -4.50 x 90e. +0.50 +2.00 x18

Jawab : B

26. Streak retinoscopy of working distance of 50 cm . the horizontal axis is neutralized first with a spherical lens -4.00 D and then the vertical axis is neutralized with cylindrical lens +2.00 D what is the refraction error

a. -4.00 -2.00 x 180b. -1.00 +2.00 x 90c. -6.00 +2.00 X90d. +4.00 -2.00 x 90e. -6.00 +1.00 x180

Jawab : A

Page 28: Contoh Soal Baru

Question no 7-12 Object is placed in front of +3.00 D lens. The lens +3.00 D in turn is position 50 cm in front of a +2.00 D lens.

27. Wher does the 3 D lens from intermediate image ?a. At optical infinityb. In front of the lensc. 0,5 m in front of the lensd. 2 m in front of the lense. 1 m in front of the lens

Jawab : A

28. Describe the intermediate imagea. Inverted, real, magnifiedb. inverted, real, minifiedc. inverted, virtual, magnifiedd. inverted, virtual, minifiede. upright, virtual, minified

jawab : C

29. what is the size of the intermediate image as compare to the object?a. Indeterminateb. On fourth the suzec. Half sized. Same sizee. Twice size

Jawab : E

30. what is the location of the final image ?a. 1 m in front of the second lensb. 0,5 m in front of the second lensc. 0,25 in front of the second lensd. 1 m behind the second lense. At optical infinity

Jawab : C

31. Seorang wanita, 56 tahun mengalami ptosis kelopak mata kanan sebesar 4 mm dan 1 mm retraksi kelopak mata kiri atas. Dia mempunyai lid crease yang tinggi dan fungsi levatornya baik pada kedua mata. Pilihan terapinya adalah :

a. Reseksi tarsoconjungtiva interna (Fasanella –Servat operation) pada kelopak atas mata kananb. Resesi levator kelopak atas mata kiric. Levator aponeurosis advancement pada kelopak atas mata kanand. Pendekatan posterior, menggunakan mullerectomy standad pada kelopak atas mata kanane. Suspensi otot frontalis pada kelopak atas mata kanan menggunakan silikon agar dapat dinilai kembali paska

operasi

Jawab : C (AAO 2006-2007 section 7 Orbit, Eyelid, lacrimal system hal 220-223)

Page 29: Contoh Soal Baru

32. Seorang laki-laki usia 25 tahun mendapatkan trauma tajam pisau yang mengenai kelopak atas mata kanannya 1 jam sebelum ke rumah sakit. Pernderita sadar dengan GCS 4-5-6, dari pemeriksaan didapatkan laserasi sepanjang 15 mm dan 12 mm terletak diatas garis bulu mata. Terdapat ptosis 7 mm pad mata kanan dan sedikit bengkak. Setelah dilakukan evaluasi dengan tepat pada bola mata, terapi terbaik yang diberikan adalah:a. Jahit robekan kulit dan tunggu beberapa bulan agar ptosisnya membaikb. Jaga luka tetap bersih dan tunggu bengkaknya berkurang dalam 2-3 hari untuk dilakukan tindakanc. Jahit lapisan otot dan lapisan kulit secara terpisahd. Eksplorasi luka untuk memeriksa aponeurosis levator dan usahakan untuk menempelkan kembali jika

mengenai tarsus setelah itu lapisan otot dan kulit dijahit.e. Jahit laserasi kulit dan lekatkan kelopak mata ke alis untuk beberapa hari agar mempercepat penyembuhan

Jawab : D (AAO 2006-2007 section 7 Orbit, Eyelid, lacrimal system hal

33. The most important determinant in selecting a corrective procedure for any type of ptosis is :a. Vertical height of the palpebral fissureb. Age of patientc. Amount of levator functiond. Duration of ptosise. Position of the upper eyelid margin relative to corneal limbus

Jawab : C (AAO 2006-2007 section 7 Orbit, Eyelid, lacrimal system hal 221)

34. Pertanyaan di bawah ini yang benar mengenai obstruksi Duktus Nasolakrimal kongenital :1. Sebagian besar obstruksi terbuka spontan 4 – 6 minggu setelah lahir2. Segera dilakukan probing3. Biasanya disebabkan gangguan pada katub Hassner4. Turbinate infracture dilakukan rutin pada bayi usia 3 bulan

Jawab : B (AAO 2006-2007 section 7 Orbit, Eyelid, lacrimal system hal 261-263)

35. Seorang wanita usia 70 tahun datang dengan keluhan kedua matanya sering berair. Pada pemeriksaan didapatkan entropion, trichiasis dan dilakukan tes snap back dengan hasil positif. Beberapa tanda klinis yang menunjukkan disinsersi pada retraktor adalah:1. ptosis palpebra inferrior2. forniks inferior lebih dalan daripada normal3. terdapat sedikit atau tidak ada pergerakan kelopak mata bawah pada saat penderita melihat ke bawah4. terdapat retraksi otot orbikularis

Jawab : A (AAO 2006-2007 section 7 Orbit, Eyelid, lacrimal system hal 202)

36. Ligamentum transversa superior adalah :a. Ligamentum Lockwood b. Ligamentun Soomeringc. The ROOFd. Ligamentum Whitnall’s

Jawab : D (AAO 2006-2007 section 7 Orbit, Eyelid, lacrimal system hal 142)

Page 30: Contoh Soal Baru

37. Seorang laki-laki 60 tahun datang dengan basalioma pada kelopak bawah mata kiri, dilakukan eksisi luas yang meninggalkan defek lebar (80% kelopak bawah mata kiri)> metode yang dapat digunakan untuk merekonstruksi defek adalah :

1. Semicircular flap2. Modified hughes procedure3. Cuttler beard flap4. Mustarde flap

Jawab : D (AAO 2006-2007 section 7 Orbit, Eyelid, lacrimal system hal 192)

38. Socket Anoftalmia yang dapat diterima secara fungsional dan kosmetika seharusnya adalah :1. Protesa nyaman dan terlihat menyerupai mata normal2. Palpebra tampak normal dan tonus adekuat untuk mensuport protesa3. volume implant orbita mencukupi di tengah orbita orbita4. Pergerakan transmisi implan baik

Jawab : E (AAO 2006-2007 section 7 Orbit, Eyelid, lacrimal system 121)

39. Dalam menangai laserasi full thickness kelopak mata atas akibat trauma yang mengenai tepi kelopak mata , seorang oftamologis sebaiknya mengerti struktur anatomi kelopak mata yang normal. Berdasarkan urutannya :a. Kulit, preaponeurotic fat, septum, otot orbicularis, aponeurosis levator, otot Muller’s, conjungtivab. Kulit, otot orbicularis, preaponeurotic fat, septum, otot Muller’s, aponeurosis levator, conjungtivac. Kulit, preaponeurotic fat, otot orbicularis, septum, aponeurosis levator, otot Muller’s, conjungtivad. Kulit, otot orbicularis, septum, preaponeurotic fat, aponeurosis levator, otot Muller’s, conjungtiva

Jawab : D (AAO 2006-2007 section 7 Orbit, Eyelid, lacrimal system hal 139-140)

40. Epicantus is medial canthal fold that may result from immature midfacial bones or a fold of skin and subcutaneus tissue. The type of epicantus if the fold is most prominent in the lower eyelid :

a. Epicantus tarsalisb. Epicantus palpebralisc. Epicantus supraciliarisd. Epicantus inversus

Jawab : D (AAO 2006-2007 section 7 Orbit, Eyelid, lacrimal system : 156)

Soal lain tentang :

1. Duane syndrome2. Intermitten exotropia3. Refractive accomodative esotropia4. Monofixation syndrome5. 6th nerve palsy6. Essential infantile esotropia7. Operasi exotropia tanpa overaksi otot oblique

40

30 30 30

20

8. Visual confusion9. Mobious syndrome

Page 31: Contoh Soal Baru

10. Primary action inferior oblique 11. Corneal neovascular pada contact lensa (hal 206)12. Terapi Bandage Contact lens (hal 202)13. Keuntungan RGP (hal 191)14. Ortho keratology (203)

KOI Board Examination Questions from Andalas UniversityPadang, West Sumatra

1. A healthy 26 years old man came to Ophthalmology clinic with sudden blurred vision, micropsia, and decreased color vision. He’s already had the same condition before. The patient was fired from his job one month ago. Which is the treatment for the patient:

A. Observation and bed rest.B. Oral and Systemic Corticosteroids.C. Topical Midriatic.D. Carbonic Anhidrase Inhibitor.E. Neurotrophic vitamins.

(Answer: D) (AAO 2005 Section 12, pg 51)

2. A 20 years old man came to an Ophthalmology clinic in St. Louis, Ohio. He complained of visual loss, metamorphosia, and paracentral scotomata since he started working in a chicken farming. Other probable signs of his ophthalmic signs, except:A. Punched-out chorioretinal lesions.B. Juxtapapillary atropic pigmentary changes.C. Vitritis.D. Choroidal neovascularizationE. Could not be determined from given information (Answer: C) (AAO 2005 Section 12, pg 79)

3. A 56 years old uncontrolled diabetic woman with history of diabetes for 15 years, complained of severe blurred vision since the last three years. She had history of hospitalization twice and insulin-dependent before meal. Her fundus examination shown moderate neovascularization elsewhere with vitreous hemorrhage. The best treatment as DRS recommendation because of its highest risk of Severe Visual Lost, for the patient is

A. Pan Retinal photocoagulation treatment.B. Early surgical vitrectomy.C. Fundus examination in the next 6 months.D. Control of blood glucose.E. Control of kidney functions as soon as possible.

Page 32: Contoh Soal Baru

(Answer: A) (AAO 2005 Section 12, pg 109)4. A 39 years old woman was referred from Department of Internal Medicine to the

Department of Ophthalmology with uncontrolled blood glucose level. Her fundus examination showed PDR. This could be classified as High-risk PDR because of:1. She’s in premenopause age2. Presence of moderate neovascularization with vitreous hemorrhage3. She has vitreous hemorrhage after severe blunt ocular trauma4. Presence of mild neovascularization of the disc with vitreous hemorrhage. (Answer: C 2 & 4) (AAO 2005 Section 12, pg 109)

5. A 9 years old boy came with complains of intermittent itching on both eyes that diminished if she was in cold weather condition, copious mucous discharge. Hisexamination shown diffuse papillary hypertrophy, serpiginous corneal ulcer. The patient is should be treated with:1. Tear film artificial2. Topical corticosteroid3. Mast cell stabilizer4. Cyclosporine (Answer: B 1 & 3) (AAO 2005 Section 8, pg 197)

6. A 48 years old woman with history of recurrent red eye on her right eye, tearing, pain, photophobia and phthisis bulbi on the left eye. Her examination shows ulcer in right corneal periphery. Her laboratory finding revealed parasitemia. The effective treatment for this patient is

A. Contact lens + oral corticosteroidB. Topical corticosteroid + contact lensC. Conjunctival flap + topical corticosteroidD. Amnion graft + conjunctival flap + topical cyclosporineE. Amnion graft +bare sclera +topical cyclosporine

(Answer: E) (AAO 2005 section 8, pg 221)

7. A 40 years old woman came with complains of blurred, red, and pain in right eye, with history of cataract surgery with IOL implantation in 10 days ago. Preoperative condition that had high risk of triggering this condition, are:1. Severe chronic blepharitis.2. Lachrymal drainage abnormalities3. Cicatrical conjunctivitis4. Corneal cicatrix (Answer: A 1, 2, & 3) (AAO 2005 Section 8, pg 214)

8. A 12 years old boy complains of proptosed right eye, without pain or blur. His ophthalmic examination shows axial proptosis, VA 20/20 ODS. CT scan revealed isodent intraconal mass. Recommended treatment for this patient:A. Observation onlyB. Surgical excisionC. Biopsy in order to determine etiologyD. ChemotherapyE. Radiation therapy. (Answer: A) (AAO 2005 Section 7, pg 74)

Page 33: Contoh Soal Baru

9. A 25 years old man with history of post cranial trauma complains of blurr eye. His CT Scan examination revealed fracture of orbital floor. Treatment for this patient are:

1. Oral steroids 2. Systemic antibiotics 3. Observation for 7-10 days4. Surgical (Answer: B 1 & 3) (AAO 2005 Section 7, pg 104)

10. Surgical indication for above case (question number 9) are:1. Enophthalmus exceeding 2 mm2. Large fracture involving at least half of the orbital floor3. Diplopia with limitation of up gaze and/or down gaze within 30o of the primary

position4. Orbital hemorrhage (Answer: A 1, 2 & 3) (AAO 2005 Section 7, pg104)

11. A 47 years old man with scar on his left forehead. In the ophthalmology examination, VA 5/15, fine keratic precipitate, flare and cell was negative in anterior chamber, corneal sensibility was decreased. The most probable diagnosis for this patient is:A. Keratitis HZOB. Keratitis HSVC. Keratouveitis HZO.D. Keratouveitis HZV.E. Viral Anterior Uveitis. (Answer: C) (AAO 2005 Section 8, pg 147)

12. A 5 years old girl was consulted from Pediatric Department with intermittent red eye and joint sickness. There also lymphadenopathy and splenomegaly. In eye examination was found blurred vision, lens opacity, red eye, corneal edema RE, band keratopathy, and anterior uveitis in both eyes. X-ray examination revealed bamboo’s appearance. In the patient, management that not be recommended:A. Topical and systemic corticosteroidsB. Topical and systemic antiglaucomasC. Topical and systemic NSAIDsD. Cataract extraction and IOL implantation in REE. Scrapping or chelation with sodium EDTA in both eyes (Answer: D) (AAO 2005 Section 9, pg 143)

13. A 20 years old man was consulted from Internal Medicine Department with redness in his right eye. His hemoglobin was 3 mg %, CD4 12, Ig G antitoxoplasma was increased, thoracic X-ray of TB duplex. His fundus examination shown massive hemorrhage. The diagnosis of the patient isA. CMV retinitisB. Posterior uveitisC. Anemia.D. ToxoplasmaE. CRVO. (Answer: C) (AAO 2005 Section 9, pg 242)

Page 34: Contoh Soal Baru

14. A 39 married woman came with red and painful on the right eye since 1 week ago. In eye examination: VA light perception, red eye, palpebral margin edema, anterior chamber: flare (+), membrane(+), occlusion pupil, lens not clear, USG shows vitreous opacity, history of trauma was unclear. The patient complained mild deafness. Laboratory finding: Mantoux test 14 mm, X ray examination revealed TB duplex. The most probable diagnosis of the patient is:A. Anterior uveitisB. EndophthalmitisC. Behcet syndromeD. Vogh Koyanagi Harada syndromeE. Sarcoidosis (Answer: B) (AAO 2005 Section 9, pg 207)

15. A 19 years old man, who’s a contact lens wearer for 4 months only on the right eye, came to Ophthalmology clinic, complained itching, pain, decreased visual acuity. In eye examination shown red eye, mucous tear film, papil hypertrophy of tarsal conjunctiva. The most probable diagnosis of the patient was based on the:A. Hypersensitivity type 1B. Hypersensitivity type 2C. Hypersensitivity type 3D. Hypersensitivity type 4E. Hypersensitivity type 5 (Answer: A) (AAO 2005 Section 8, pg 186)

16. A 70 years old man came with complain of blurring of RE, headache with nausea and vomiting. Eye examination shown IOP 62 mmHg, hyperemic conjunctiva, corneal edema, narrow anterior chamber. LE was normal, with narrow anterior chamber. Most common mechanism of etiology:

A. Lens luxation.B. Pupillary block.C. PAS.D. Aqueous misdirectionE. Anteriorly iris caused by tumor (Answer: B) (AAO 2004 Section 10, pg 101-102)

17. Medical treatment that should not be given to the patient (question number 16) is:A. Pilocarpine 1%-2%B. Timolol maleateC. Carbonic anhidrase inhibitorD. Manitol 20 %E. Apraclonidine (Answer: E) (AAO 2004 Section 10, pg 101-102)

18. To determine whether patient’s trabecular angle was synechia or apposition (question number 16), it’s better to examine him with the type of gonioscopy:A. Goldman type goniolensB. Zeiss type goniolensC. Koeppe type goniolensD. Barkan type goniolensE. Wurst type goniolens

Page 35: Contoh Soal Baru

(Answer: B) (AAO 2004 Section, pg 72)

19. A 24 years old man came with blurring and pain of RE. Eye examination shown IOP 53 mmHg, hyperemic conjunctiva, corneal edema, flare (+) in anterior chamber. Gonioscopy revealed normal angle with khaki colored cell in inferior trabeculum. Two months ago, patient had ocular trauma that causing vitreous hemorrhage. The right diagnosis of the patient is:A. Ghost cell glaucoma.B. Hemolytic glaucoma.C. Angle recess glaucomaD. Lens induced glaucomaE. Glaucomatocyclitic crisis (Answer: C) (AAO 2004, Section, pg 95-98)

20. A 55 years old woman had benign tumor operation of inferior palpebral margin, mass dimensions was > 50 %. In the palpebral reconstruction, anterior lamellar can be taken from pre auricular or retro auricular skin. This method is known as:A. Modified Hughes procedureB. Mustard procedureC. Tenzel Flap procedureD. Semicircular Pedicle FlapE. Cuttler-Beard Procedure (Answer: A) (AAO 2005 Section 7, pg 192)

21. A 25 years old woman came to Ophthalmology clinic with complain of proptosed of LE since 1 month ago. LE became more proptosed since she was pregnant. CT Scan shown encapsulated homogenous mass. Recommended treatment for the patient was excisional. The right diagnosis for the patient:A. Hemangioma cavernousB. HemangiopericytomaC. LymphangiomaD. Arteriovenous fistulaE. Orbital varices (Answer : A) (AAO 2005 Section 7, pg 67)

Page 36: Contoh Soal Baru

SOAL – SOAL unhas 2008

Seorang pasien umur 71 tahun datang ke poliklinik mata dengan keluhan visus menurun

secara perlahan-lahan yang disertai adanya gangguan penglihatan sentral.

Segmen anterior bola mata relatif normal. Pada pemeriksaan funduskopi didapatkan

adanya lesi berbentuk bulatan kecil-kecil berwarna kekuningan pada level RPE di area

makula. 1.Diagnosis yang paling tepat pada kasus diatas adalah :

a.Non neovascular AMD

b.Neovascular AMD

c.CME

d.Pattern dystrophy of RPE

e.CSCR

Jawaban : A (AAO section 12, hal 55-87)

2.Pemeriksaan pendukung yang paling utama yang seharusnya dilakukan pada pasien

tersebut (skenario 1) diatas adalah :

    a.FFA

    b.ERG

    c.Color blindness test

    d.ICG

    e.OCT

    Jawaban : A (AAO section 12, hal 55-87)

Seorang wanita umur 47 tahun datang ke poliklinik mata dengan keluhan gangguan

penglihatan terutama pada senja hari. Bapak dan saudara laki-lakinya juga menderita hal

yang sama. Walalupun segmen anterior bola mata tidak ditemukan kelainan akan tetapi

terlihat gambaran pigmen-pigmen hitam tersebut di empat kuadran mid perifer retina.

3.Pemeriksaan pendukung untuk penegakan diagnosis yang tepat pada kasus diatas

adalah:

    a.ERG dan visual field testing

    b.ERG dan FFA

    c.Visual field testing dan FFA

   d.ERG dan Ishihara color testing

Page 37: Contoh Soal Baru

    e.FFA dan Ishihara color testing

    Jawaban : A (AAO section 12, hal 203-212)

4. Buta senja (nictalopia) yang dirasakan oleh pasien tersebut diatas oleh karena adanya

kerusakan fungsi retina pada level :

    a.Sel-sel RPE

    b.Sel-sel photoreseptor

    c.Sel-sel ganglion

    d.Lapisan serabut syaraf

    e.Sel-sel cone di area makula

    Jawaban : B (AAO Section 12, hal 207)

5. A 40-year-old woman present with a progressively enlarging clear cystic mass along the

eyelid margin.The  most likely diagnosis is

    a.Epidermal inclusion cyst

    b.Apocrine hidrocystoma

    c.Syringoma

    d.Trichofolliculoma

    e.Basal cell carcinoma

    Jawaban : B

6. Seorang perempuan 74 tahun dengan riwayat nyeri, pembesaran massa yang

progressif di bagian sentral palpebra superior selama 2 tahun. Didapatkan distorsi

pada margo palpebra dan madarosis. Diagnosis yang paling tepat adalah

    a.Glandula sebaceus carcinoma

    b.Squamos sell carcinoma

    c.Amelanotic melanoma

    d.Basal cell carcinoma

    e.Dermal nevus

    Jawaban : D

7. Enam jam setelah dilakukan blepharoplasty bilateral, pasien mengeluh tiba-tiba nyeri

pada mata kanan. Setelah jahitan dilepas, tampak palpebra bengkak dan ekimosis.

Page 38: Contoh Soal Baru

Langkah pertama yang tepat adalah :

    a.Membuka luka untuk mengelimansi kemungkinan retrobulbar hemorrhagic

    b.Memikirkan kemungkinan trombosis sinus cavernosis dan uji sensitifitas kornea

    c.Mengukur visus dan uji respon pupil

    d.Mulai pengobatan dengan kompres es

    e.Mulai pengobatan dengan kompres hangat

    Jawaban : C

8. Anak 14 tahun dengan keluhan lakrimasi dan sekretd ari mata kanan, dialami sejak

lahir. Dimana pernyataan yang paling tepat di bawah ini :

    a.Dye disappearance testing menunjukkan tidak adanya asimetris

    b.Kondisi ini dapat sembuh spontan

    c.Terapi yang diutamakan berupa probing ductus nasolakrimal

    d.Abnormal pungtum bisa menjadi penyebab

    e.Perlu dilakukan Jones 1 dan 2 untuk menegakkan diagnosis

    Jawaban : C

9. Seorang laki-laki 30 tahun, dengan trauma terkena pisau pada palpebra superior mata

kanan, dialami 1 jam MRS, pasien sadar dan ditemukan laserasi sepanjang 15mm dengan

lokasi kira-kira 12 mm diatas silia. Pasien mengalami ptosis 7 mm di bagian kanan dan

edema minimal. Sesudah dilakukan pemeriksaan bola mata,terapi yang terbaik adalah

    a.Perbaikan laserasi kulit dan menunggu beberapa bulan untuk perbaikan ptosis

    b.Menjaga kebersihan luka dan menunggu proses inflamasi mereda dalam 2-3 hari

    c.Perbaikan sebagian lapisan otot dan kulit

d.Eksplorasi luka untuk memeriksa levator aponeurosis dan menempatkan kembali

jika terlepas dari tarsusnya,sesudah perbaikan kulit dan otot

e.Perbaikan laserasi kulit dan palpebra beberapa hari untuk merangsang penyembuhan.

Jawaban : D

10. Pernyataan berikut yang salah mengenai Rhabdomyosarcoma adalah :

a.    Rata-rata umur 7-8 tahun

b.    Progresif proptosis unilateral

c.    Gejala yang muncul adalah: palpebra edema dan perubahan warna, ptosis,

Hifema, Strabismus

Page 39: Contoh Soal Baru

d.    Kadang-kadang ada riwayat trauma

e.    Dapat metastase ke kelenjar limfa cervical dan preaureculer

Jawaban : E

11. Yang termasuk faktor resiko terjadinya Basal Cell Ca adalah :

1.Kulit kuning langsat, mata biru, rambut merah

2.Usia pertengahan ke atas

3.Exposure sinar matahari

4.Merokok

Jawaban : E

12. Tanda-tanda okuler pada tumor limphoma maligna adalah:

a.    Specifik uveitis

b.    Umumnya bilateral

c.    Retinal vasculitis

d.    Vascular oclution

e.    Exudative retinal detachment

jawaban : A

13. Berdasarkan klasifikasi sitologi dari melanoma uvea, yang mempunyai

prognosis paling jelek adalah :

a.Spindle cell nevus

b.Spindle Cell Melanoma

c.Epithelioid melanoma

d.Mixced cell type

14. Seorang pria Jepang, 41 tahun dengan riwayat trauma tumpul mata dengan

visus yang baik dan tidak ada riwayat operasi sebelumnya. Keluhan saat ini terjadi

penurunan visus dan nyeri berat pada kedua mata. Terdapat bilateral uveitis,

alopecia, vitiligo dan kelainan serebrovaskular. Terdapat juga Retinal Detachment

pada satu mata. Diagnosa yang tepat adalah

    a. Sarkoidosis

    b. Simpatetika oftalmika

    c. Vogt-Koyanagi-Harada Syndrome

    d. Bechet Syndrome

Page 40: Contoh Soal Baru

    Jawaban : C

15. Seorang perempuan umur 67 tahun dengan uveitis moderate dan vitritis dan

subretinal infiltrat.Kondisi ini memberikan respon minimal terhadap topical

kortikosteroid. Pasien merasa lemah dan pusing, test apa yang penting dilakukan :

    a. Gallium scan

    b. Westergren sedimentation rate dan C-reaktif protein

    c. PPD dan chest x-ray

    d. CT scan atau MRI kepala

    Jawaban : D

16. Organisme mana dibawah ini yang paling sering menyebabkan endoftalmitis ec

trauma tetapi jarang menyebabkan endoftalmitis ec operasi katarak atau bleb

related endoftalmitis :

    a. Stap. Epidermididis

    b. Stap. Aureus

    c. Hemophyluss influensa

    d. Bacillus Sereus

    Jawaban : D

17. Seorang pasien dengan post trabekulektomi disertai mitomisin C, didapatkan

severe bleb-related endoftalmitis. Visus awal 20/20 dan sekarang menjadi 1/300.

Dimana pernyataan dibawah ini yang tidak tepat :

    a. Prognosis visus jelek

b. Organisme yang berhubungan dengan bleb-related endoftalmitis

adalah Hemofilus influensa atau Streptococcus.

c. Oleh karena visus lebih baik dari 1/~ diperlukan kultur vitreus dan

injeksi antibiotik

d. Endoftalmitis bisa timbul berbulan-bulan sampai bertahun setelah

operasi filtering glaukoma

    Jawaban : C

18. Seorang laki-laki kulit putih, 80 tahun dengan keluhan visus mata kanan

menurun, nyeri dan hiperemi konjungtiva. Pada pemeriksaan ditemukan TIO 45

Page 41: Contoh Soal Baru

mmHg dengan adanya sel dan flare tanpa keratit presipitat. Terdapat juga katarak

yang padat dan glaukoma sudut terbuka. Diagnosis yang paling tepat adalah

    a. Phacolitik glaukoma

    b. Phacoanafilaksis

    c. ICE sindrome

    d. Fuchs heterochromic iridosiklitis

    Jawaban : A

19. Pada “Persistent Corneal Epithelial Defect”

terdapat hal-hal berikut KECUALI :

    a. Defek kornea yang “chronic non healing epithelium”

    b. Hipostesi kornea

    c. Neovaskularisasi dan kekeruhan kornea

    d. Prediksi pada superior atau superior kornea

    e. Dapat terjadi perforasi kornea

    Jawaban D (BSC 8 hal 96)

20. Phaco emulsification for cataract case with limited zenular support, the following

factor must be remember :

    1. Reducing flow rate

    2. Insertion capsular tension ring

    3. Lowering the bottle height

    4. Increasing power ultrasound

    Jawaban : A (BCSC sec 11 hal 215)

21. A child with anisometropic should be given:

    A. Full cycloplegic refractive the two eyes

    B. Face uncycloplegic refractive the twol eyes

    C. Under correction for the hight myopic eye

    D. Under correction for the hight myopic eye

    E. Over correction for lower myopic eye

    Jawaban : A (BCSC sec 3 hal 150)

Page 42: Contoh Soal Baru

22. Eye of the patients have not image magnification of minification like emetropic

eye:

    A. Hyperopic eye with contact lens correctin

    B. Hyperopic eye with convex lens spectacle

    C. Myopic eye with contact lens correction

    D. Myopic eye with concave lens spectacle

    E. Myopic pseudophakia with concave lens spectacle

    Jawaban : E (BCSC sec 3 hal 209)

23.    Retinal  image of contact lens is influenced by :

a. Power of contact lens

b. Diameter of contact lens

c. Refractive status of patients

d. Verses distance of contact lens

e. Anterior surface curvature of contact lens

Jawaban : D (BCSC sec 3 hal 173)

24. To reduce aniseikonia of two eyes, can be achived by the following may,

exception :

    a. Miopic phakic eye be corrected by contact lens to increase mirage size

    b. Miopic phakic eye be corrected by contact lens to decrease mirage size

    c. Miopic phakic eye be corrected by spectacle lens to increase mirage size

    d. Hyperapic phakic eye be corrected by spectacle lens to decrease mirage

size

    e.  Hyperapic phakic eye be corrected by contact lens to increase mirage size

    Jawaban : A (BCSC sec 3 hal 175)

25.Tranposition  of the lens  S – 3 = C + 3 x 70:

    a. S + 3 = C + 3 x 160

    b. S – 3 = C – 3 x 70

    c. C – 3 x 160

    d. C + 3 x 70

    e. S + = C – 3 x 160

    Jawaban : C (BCSC sec 3 hal 92)

Page 43: Contoh Soal Baru

26. Pemeriksaan angiografi floresin pada choroidal neovascularization

memeprlihatkan gambaran hiperfloresin pada fase ;

a.    prearterial

b.    arterial

c.    aretrivenous

d.    venous

e.    last phase.

Jawaban A.

            AAO, Sec 12, 14, 21.  AAO 2003-04.

27.    Penyebab tersering gagalnya penanganan operasi dari ablasi retina

rhegmatogen adalah karena adanya :

a.    giant retinal break

b.    proliferative vitreoretinopathy

c.    lattice degeneration lebih dari 2 kuadrant

d.    perdarahan vitreous

e.    Posterior vitrewous detachment

Jawaban : B

AAO, Sec 12,  246,  AAO 2003-04.

28.    Seorang anak umur 3 tahun, dengan kataran pada mata kiri, menunjukkan

fiksasi yang jelek dan esotropia di mata kiri. Mata kanan nampak normal. Manakah

kalimat dibawah ini yang paling tepat :

a.    Terapi ambliopia harus diberikan mengawali operasi katarak

b.    Kapsulotomi posterior tidak boleh dilakukan pada saat operasi oleh karena

dapat menyebabkan ablasi

c.    Operasi katarak dengan implantasi IOL dapat dilakukan untuk merehabilitasi

visus

d.    Implatasi IOL tidak boleh dilakukan pada anak-anak

e.    Esotropia mata kiri harus diperbaiki mengawali operasi katarak

Jawaban : C

Page 44: Contoh Soal Baru

29.    Seorang wanita, 55 tahun dengan miop dan keluhan diplopia monokular

disertai keluhan sulit mengendarai mobil pada malam hari. BCVA -2 D, Visus 20/30.

Pada pemeriksaan slit lamp terdapat nuklear sklerosis minimal. Pemeriksaan

tambahan apa yang diperlukan untuk mengevaluasi gejalanya :

a.    Topografi kornea

b.    Red refleks

c.    MRI

d.    Fluorosens angiografi

Jawaban : B

30.    Seorang laki-laki, 70 tahun dengan keluhan kabur saat mengendarai mobil.

Visus 20/70 mata kanan, dan 20/40 mata kiri. Test lapang pandang Goldman

terdapat konstriksi pada mata kanan.Terdapat juga nuklear katarak yang moderat

pada mata kanan dan yang ringan pada mata kiri. TOD 23mmHg, TOS 18mmHg.

Pasien menggunakan timolol dan dorzolamide pada mata kanan. CDR OD 0,8, OS

0,6. Pemeriksaan fundus lainnya dalam batas normal. Pernyataan yang benar

adalah :

a.    Operasi katarak disertai operasi filtrasi glaucoma adalah satu-satunya terapi

b.    Penggunaan latanoprost sesudah operasi katarak dapat meningkatkan

resiko post op CME

c.    Operasi katarak tidak dapat dilakukan karena resiko kehilangan fiksasi post

op katarak

d.    Obat-obat glaukoma harus diberikan secara maksimal sebelum operasi

katarak

e.   Konstriksi pada pemeriksaan lapangan pandang kemungkinan disebabkan

oleh glaukoma

Jawaban : B

31.Yang mana pengobatan yang paling tepat untuk bayi

tersebut :

a. mata yang terbaik fiksasinya ditutup dan diikuti

dengan operasi pada mata yang lain

b. Segera lakukan operasi pada satu mata, dan mata

Page 45: Contoh Soal Baru

yang lain pada umur 3 bulan

c. Lakukan operasi pada satu mata dan segera lakukan

koreksi afakia, lakukan operasi pada mata sebelah

sebelum umur 3 bulan

d. Observasi sampai umur 3 bulan dan lakukan operasi

apabila  timbul nystagmus

    e. Lakukan operasi pada kedua mata pada umur 3 bulan

Jawaban : C (BCSC 11, hal 195)

32. Pada bayi tadi metode operasi yang paling tepat

atau tahapan selam operasi katarak adalah sebagi

berikut KECUALI

    a. Aspirasi lensa yang katarak

    b. Implantasi posterior chamber IOL

    c. Kapsulotomi posterior dan vitrektomi anterior

secukupnya

    d. ECCE

    e. Small Incisisi Cataract Surgery

Jawaban : B (BCSC 11 hal 156)

33. Seorang laki-laki umur 60 tahun, berhasil

dioperasi nuclear sklerotik katarak pada mata kanannya

dengan fekoemulsifikasi. Dokter bedah mata secara

tidak sengaja meninggalkan viscoelastik yang cukup

banyak didalam mata. Berapa lama setelah operasi

selesai terjadi peningkatan TIO secara signifikan.

    a. 30 menit

    b. 2 jam

    c. 4 jam

    d. 10 jam

    e. 24 jam

Jawaban : C (BCSC 11, hal 172)

34. You have +10D lens dan +20D lens. You want to make

Page 46: Contoh Soal Baru

a 2x magnifier. What is the distance between the

lenses to produce this magnification?

    a. 10 cm

    b. 15 cm

    c. 5 cm

    d. 25 cm

Jawaban : C (BCSC 3,hal 57)

35. You have two +2,00 D lenses. How far apart are the

two lenses so that an object at infinity is focused 1m

to the right of the second lens?

    a. 1.0m

    b. 0.5m

    c. 1.5m

    d. 0.75m

Jawaban : C (BCSC 3, hal 57)

36. A Galielan telescope is constructed with a +5,00D

objective and a -10,00D eyepiece. What is

magnification of the telescope?

    a.  ½ X

    b. 50 X

    c. 2 X

    d. 5 X

Jawaban : C (BCSC 3, hal 74)

37. Seorang wanita 44 tahun, datang dengan keluhan

diplopia. Ptosis 4 mm, tidak ditemukan upgaze dan

adduksi, serta 50% reduksi pada abduksi. Pupil 5 mm

dan kurang reaktif, tetapi tidak terdapat RAPD. Visus

normal, pada hasil pemeriksaan eksterna dan

pemeriksaan fundus normal. Lokasi lesi yang sering

ditemukan adalah

    a. Orbit

Page 47: Contoh Soal Baru

    b. Sinus cavernosus

    c. Brain stem

    d. Neuromuscular junction

Jawaban : B

38. Sehubungan dengan pertanyaan diatas, prosedur

pertama yang terbaik adalah

    a. MRI dengan kontras

    b. CT scan tanpa kontras

    c. USG

    d. Tes Tensilon

Jawaban : A

39. MRI memberikan hasil yang lebih baik dibanding CT

scan kecuali pada keadaan dibawah ini

    a. Penyakit demielinisasi

    b. Infark akut

    c. Hemorrahagic akut

    d. Abnormal parenkim kongenital

Jawaban : C

40. Seorang wanita obesitas, 23 tahun, datang dengan

keluhan sakit kepala dan transient visual obscuration.

Terdapat papil edem bilateral. Langkah selanjutnya

adalah

    a. Dimulai dengan pemberian acetazolamide oral

    b. Pemeriksaan Lumbal puncture

    c. Menganjurkan diet

   d. Pemeriksaan CT scan dan MRI

Jawaban : D

Page 48: Contoh Soal Baru

TRY OUT BOARD UNSRAT DESEMBER 2007

1. 30 years old male came to you with redness on his left eye since 2 days ago. The redness of the eye occurred since he had trauma from a branch of three. On slit lamp examination : little conjunctival injection, gray white, dry appearing corneal infiltrate with feathery margins. Corneal scrapping reveals fusarium spp as the infectious cause. The treatment for this patient is :

a. Miconazole 1%b. Oral fluconazole 200-400 mg/dayc. Chlorhexidine topical 0,02%d. Propamidine isethionate 0,1%e. Natamycin 5%

Answer E2. A 25 years beautiful girl came to your clinic with redness of the eye for 2 months. Examination

reveals follicle in superior tarsus conjunctiva, diffuse papillary hypertrophy, artl’s line and herbert’s pits. These statements are true :

1. Isolation the causative agent with McCoy cells2. Topical tetracycline 1% twice daily for 2 months3. Resolve spontaneously in 6-18 months if left untreated 4. Oral erythromycin 500 mg 4 times a day for 3 weeks

Answer E3. A baby 3 days old come to eye clinic with complain copious purulent conjunctival

discharge, marked conjunctival hyperemia and chemosis since 2 days ago. The appropriate diagnosis for this patient are true :

a. Gonoccocal conjunctivitisb. Chlamydial conjuctivitisc. Microsporidial conjunctivitisd. Trachomae. Loisiasis

Answer A4. 54 year old woman complains of discomfort and foreign body sensation in her right eye over

the past few months. Slit lamp shows papillary hypertrophy of the upper palpebral conjunctiva with prominent vessels and hyperemia of the upper bulbar conjunctiva with punctate staining with rose bengal. Corneal changes reveal micropannus and fine filaments at the superior periphery. The most likely diagnosis is:

a. Terrien’s degenerationb. Mooren’s ulcer

Page 49: Contoh Soal Baru

c. Superior limbic keratoconjunctivitisd. Trachomae. Rheumatoid arthritis with ocular involvementAnswer C

5. Streak retinoscopy was perfomed on a 3 years japanese old boy. Working distance was 67 cm. You find with the movement sweeping the 180 meridian and against movement sweeping 90 meridian. The power cross is :

What is the correct spectacle prescription?a. +3.50-5.00x180b. -3.50+1.50x180c. -2.00 +5.00x180d. +1.50 -5.00x180e. +1.50 -5.00x90Answer D

6. A 4 year old child is referred for a new onset of bilateral epiphora. Examination shows on both lower eyelids rubbing against the inferior cornea. The parents state that similar symptoms occurred in an older sibling but resolved without treatment. What is the most likely diagnosis:

a. Epiblepharon

b. Entropionc. Euryblepharond. Trichiasise. Ankyloblepharon

Answer A

7. What is the pathophysiologic mechanism underlying the condition in above question : a.Laxity of tarsal plateb. Laxity of the canthal tendonsc.Abnormal attachment of the orbital septumd. redudancy of skin and pretarsal musclee. none of above

Answer D8. 50 year old man came to the eye clinic with the chief complain of pain on his right eye since 3

days ago, pain associated with tearing, edema and erythema within his medial canthal regio with distension of the lacrimal sac. These are true except :

a. Therapeutic probing of the nasolacrimal duct with Bowman probeb. Topical antibiotics are limited value when stasis is presentc. Aspiration of the lacrimal sac if the mucocele/pyocele is localised and pointingd. A localized abscess requires incision and drainage, the incised abscess is packed opene. Gram positive bacteria is the most common cause

Answer A9. A 60 years old woman had headache, red eye, microcystic corneal edema following cataract

extraction in the left eye 2 days ago. The treatment of choice in these case are:

+3D

-2D

Page 50: Contoh Soal Baru

a. Parasympathomimetic agentb. Corticosteroids and carbonic anhidrase inhibitorsc. Prostaglandin analoguesd. Cholinergic agonistse. Anticholine esterase agents

Answer: B.10. The diagnosis in above Question is :

a. Phacolytic glaucomab. Phacoanaphylaxis glaucomac. Phacomorphic glaucomad. Lens particle glaucomae. Ectopia lentis

Answer: D.11. A 60 years old black man came in for an eye examination. He did not have any specific

complaints. He uses only over the counter reading glasses and had never visited an ophthalmologist in the past. Visual acuity was 20/60 OU with correction. The anterior segment of both eyes was normal with the exception of early cataractous changes OU. The IOP was 30 mm Hg OU. What kind of further examination must be done to diagnose this complaints:

1. Opthalmoscopy2. Gonioscopy3. Visual field test4. Contrast sensitivity test

Answer: A.12. A 15 year old student was brought to the Clinic with a history of trauma to the left eye while

playing football. The visual acuity was hand movement OS and 20/20 OD. Examination of the left eye showed circumcorneal congestion, a moderately haze cornea, and small haemorhage in AC. The IOP was 50 mm Hg. Gonioscopy reveals angle recession. The treatment is often best accomplish with, except:

a. Timolol Maleat 0.5%b. Acetazolamidec. Laser Trabeculoplastyd. Apraclonidin HCle. Latanoprost

Answer: C.13. A 56 year-old woman presented to the eye clinic with sudden blurred vision, headaches

sometimes her eye feel mild pain and see halos. The pain and blurred vision resolve spontaneously, especially during sleep-induced miosis. She routine came to ophthalmologist with the IOP sometimes raised and sometimes normal. The visual acuity was 6/60 OD and 20/20 OS. The right eye showed circumcorneal injection, mild diffuse corneal haze, shallow anterior chamber and the pupil slow adapted to the light. The lens was early cataractous changes. The left eye was normal except for a shallow AC. The IOP was 40 mm Hg OD and 16 mm Hg OS. Gonioscopy OD revealed narrow angle and PAS at 11-12 o’clock. Gonioscopy OS revealed a narrow angle. The Diagnosis is:

a. Primary Angle-closure Glaucoma

Page 51: Contoh Soal Baru

b. Chronic Angle Closurec. Intermitten angle Closured. Primary Open Anglee. Phacomorphic glaucoma

Answer: C.

14. Treatment of choice in this case is:

a. Glycerin oralb. Pilocarpine Hcl 10%c. Timolol Maleat 0.5%d. Acetazolamide orale. Laser iridectomy

Answer: E15. A 50 year old man complaining glare especially under bright lightning condition. His near

visual acuity is worse than distance visual acuity, even after correction for presbiopia. In slit lamp examination you saw this : Your diagnosis :

Figure 1a. Anterior polar cataract b. Cortical cataractc. Posterior subcapsular cataractd. Morgagnian cataracte. Hypermature cataract

Answer : C16. If this patient were treated with topical pilocarpine, he will experience :

1. better near vision2. worse near vision3. better far vision4. worse far vision

Answer : C

17. This kind of cataract in above question can be caused by :1. Senescent2. Trauma3. Inflammation4. Topical corticosteroid use

Answer : E

Page 52: Contoh Soal Baru

18. A patient is noted to have nuclear sclerotic cataract. Cataract of this type may be associated with all the following except:

a. Early loss of near (reading) vision.b. A myopic shift.c. Monocular diplopia.d. Difficulty seeing road signs at dusk.e. Loss of far vision.

Answer A19. On pupillary dilation, a patient shows the characteristic lens change inferiorly dislocated lens

with equatorial zonular remnants. The least likely diagnosis is:

a. Marfan syndrome

b. Homocystinuria

c. Weill-Marchesani syndrome

d. Sturge-Weber syndrome

e. Hyperlysinemia

Answer D

20. A 25 years old man came to the eye clinic. Ophthalmologic examination revealed a golden brown discoloration of Descemet’s membrane around the periphery of the cornea, and sunflower cataract. The predisposing disease is :

a. Galactosemiab. Myotonic dystrophyc. Hypocalcemiad. Hepatolenticular degeneratione. Infrared radiation

Answer D

21. A 65 year old man complains of difficulty driving because of reduced vision. His best- corrected visual acuity is 20/70 OD and 20/40 OS. Goldman visual fields are constricted, more in the OD than in the OS. A moderate nuclear cataract is present OD, and mild one is seen OS. His IOP is 23 mm Hg OD and 18 mm Hg OS. He uses timolol ½ % bid OD and Dorzolamide tid OD. His cup- disc ratio is 0.8 OD and 0.6 OS. Which of this following statement is true?

a. Cataract surgery should not be considered because of the risk of loss of fixation postoperatively

b. Cataract surgery combined with glaucoma filtering surgery is the only approach that should be considered for this patient

c. The use of latanaprost after cataract surgery may increase the risk of postoperative CME

d. Staged procedures is a cataract surgery followed by filtering surgery e. The visual field constriction in this case is probably caused by glaucoma

Answer: C.22. A two weeks old baby boy came with bilateral congenital cataract. The TORCH examination is

negative. The proper time for the cataract extraction is :

a. under 3 month for the first eye, and 1 month later for the second eye

above 3 month for the first eye, and 2 weeks later for the second eye

Page 53: Contoh Soal Baru

b. under 3 month for the first eye, and 2 weeks later for the second eye

c. above 3 month, together both eyes

d. under 1 year for the first eye, and 1 month later for the second eye

Answer: C

23. Two years after the operation was done, his parents want to correct their baby’s aphakic condition. The ophthalmologist suggests secondary IOL implantation. Which is the true statement :

a. Fixation in the sulcus is preferable to fixation in the capsular bagb. Use the + 20 D IOLs, without calculate the intraocular powerc. Anterior chamber IOLs are generally recommendedd. There is an improved of binocularity when compared with contact lensese. Bilateral lens implantation surgery in children above 2 year of age is still controversial

Answer D.

24. Post operative, this baby should receive one of these, except :

a. Fluometholone and Scopolamine 0,25% for about 1 month

b. Prednisolone and Phenilephrine 0,1% for about 1 month

c. Hydrocortisone and Cyclopentolate 1% for about 1 month

d. Betamethason and Atropine 1% for about 1 month

e. Dexamethason and Cyclopentolate 2% for about 1 month

Answer B.

25. A 25-year-old woman notes the acute onset of painless photopsias and a blind spot in the temporal field of her left eye. Her visual acuity and color vision are normal. On visual field examination, her left blind spot is three times the normal size. Fundus examination shows multiple, faint, tiny spots in the posterior pole OS. The examination is otherwise entirely normal. Examination 3 days later shows the spots to be gone. Which of the following diagnosis is the most likely?

a. Pseudotumor cerebrib. Acute posterior multifocal placoid pigment epitheliopathyc. Acute zonal occult outer retinopathy (AZOOR)d. Optic neuritise. Canthaxanthine maculopathy

Answer: C

26. A 24-year-old man complains of episodes of blurred vision with flashing lights that progress over approximately 30 minutes. There are followed by a throbbing headache and resolution of his visual aymptoms. A complete ophthalmologic evaluation, including visual fields, is normal. The most likely diagnosis is

a. Cluster headacheb. Classic migrainec. Transient ischemic attackd. Superior oblique myokymia

Answer B

Page 54: Contoh Soal Baru

27. A 8 years old girl presents with a left upper eyelid edema and erythema. According her mother this condition has been presents for 2 days without history of trauma. On examination revealed her vision 20/20, pupillary reaction was normal, proptotic is not presents and no limitation of ocular motility. Pain on movement of the globe is absent. The most likely diagnosis is :

a. Orbital cellulitisb. Preseptal cellulitesc. Idiopathic orbital inflammationd. Rhabdomyosarcomae. Carotid cavernous sinus fistula

Answer B 28. Which of the following statement about above condition is false:

a. Commonly occurs in children b. The most frequent cause is ethmoidal sinusitis c. Imaging studies should be performed d. Respond to single antibiotic theraphy e. Involvement of the orbital apex

Answer E29. A 70 years old actrist presents complaining of droopy eyelids, superior fields defect and

difficulty for reading. Examination reveals pupillary reaction was normal and marginal reflex distance is found to be 1 mm. A tensilon test was negative. The most likely diagnosis is :

a. Myastenia gravis b. Dermatochalasis c. Cranial nerve III palsy d. Blepharoptosis e. Horner’s syndrome

Answer D30. Which of the following in the clinical evaluation is most significant to compare acquired

type than congenital type of diagnosis in above question ?

1. Upper eyelid crease 2. Levator function 3. Eyelid displacement on downgaze 4. Palpebra fissure height

Answer A

31. What is pathophysiologic mechanism underlying the condition in above question ? a. Muscle dysfunction b. Neurogenic pathology c. Aponeurotic dysfunction d. Redudancy of skin e. Orbital fat prolapse

Answer C32. A 9-month-old baby girl present to the eye clinic for crossed eye. She seems always to have

Page 55: Contoh Soal Baru

both eye crossed inward, all of the time, regardless of whether she is looking at near or at distance. Birth history was normal, she is in good physical health, with fix and follow behavior, normal anterior and posterior segment of both eyes. The corneal light reflex test shows equal and a cover test show no movement. The eyes appear to be crossed inward, with less sclera showing in the nasal portions of the eyes as compared with the temporal portions. The most likely diagnosis is:

a. Infantile esotropiab. Incomitant esotropiac. Pseudoesotropiad. Basic esotropiae. Sensory deprivation esodeviation

Answer C33. You are asked to see a 5-year-old little girl with amblyopia. How do you most likely manage

her ?

1. Obstacle removal2. Refractive correction3. Occlusion and optical degradation4. Kestenbaum-Anderson Procedure

Answer A

34. A mother complains that her 1-year-old boy sometimes has a visual inattention. His left eye first noticed by his mother as a glint, white to cream colored. The first examination notes leukocoria on his left eye. The least likely differential diagnosis of this boy is:

1. Persisten hyperplastic primary vitreus2. Chorioretinal colobomas 3. Retinoblastoma4. Brushfield spots

Answer D35. A healthy 30 yo man presents with a 1-week history of pain, redness and floaters in his left eye.

Examination reveals VA of 20/25 OS. Significant finding on examination are mild ciliary injection, fine keratic precipitates, mild anterior chamber cell, clear lens, and moderate anterior vitreous cell. The posterior pole appears normal except for a slightly hazy view secondary to vitreous debris. The lesion shown in the peripheral retinal. What is the most likely diagnosis ?

A. Cytomegalovirus retinitis (CMV)B. Toxoplasmic retinochoroiditisC. Endogenous bacterial endophthalmitisD. Behcet syndromeE. Acute retinal necrosis (ARN)Answer E.

36. The most appropriate initial treatment for the patient in above question is :A. Intravenous acyclovir for 10-14 day follow by oral valacyclovir, famcyclovir or

acyclovir for up 3 monthsB. Intravenous ganciclovir induction for two weeks, followed by daily maintenance

ganciclovirC. Oral prednisone 60mg to 80 mg perdayD. Erythromycin (500mg qid) or ciprofloxacin (750mg bid) E. Laser photocoagulation to demarcate the borders of retinal necrosis

Page 56: Contoh Soal Baru

Answer A.37. An 40 yo man was playing card when he noticed the vision become ‘dim’ in his left eye. He

felt well otherwise and had no pain. After a few minutes, he told his companions, and they brought him to an emergency room. Vision was limited to light perception OS, but OD was 20/25. There was a left afferent papillary defect . The fundus OS showed narrowed vessels with boxcarring. Most of the retina was abnormally pale, and the macula showed a cherry red spot. Which statement is correct for this diagnosis ?

1. Fluoresencein angiography can also be helpful in diagnosis because many patients exhibit a phenomenon dark choroids2. Irreversible damage to the sensory retina occurs after 90-100 minutes of complete disease3. Other treatments advocated include inhalation therapy with 25% oxygen-65% carbon dioxide mixture 4. It is often caused by atherosclerosis-related thrombosis occurring at the level of the lamina cribosa.

Answer C.

38. A 40 yo man was reffered to ophthalmologist from a neurologist with clinical findings

cerebellar ataxia,polyneuropathy,anosmia,deafness. An ECG examination showed arrythmias.

He was reffered because prior of those disorders, he complain about difficulty to see in dusk.

In the examination retinal athrophy and spiculed pigmentary degeneration was founded. The

diagnosis of this condition :

a. Mucopolysaccharidosesb. Tay-sachs diseasec. Kearns-sayre syndromed. Refsum diseasee. Bardet-Biedl syndrome

Answer D

39. A 30 years old pregnant women was suspected with ocular toxoplasmosis. The anterior segment was normal with whitish yellow, slightly raised, fuzzy lesion with a chorioretinal scar on funduscopic examination. IgM antibody titer for toxoplasma was positive. Right now, she is 5 months pregnant. What is the treatment for this patient?

a. Pyrimethamine & Sulfonamideb. Clindamycinc. Spiramycin & sulfadiazined. Atovaquinee. Spiramycin & Pyrimethamine

Answer C

Page 57: Contoh Soal Baru

40. A 40 yo man was reffered to ophthalmologist from a neurologist with clinical findings

cerebellar ataxia,polyneuropathy,anosmia,deafness. An ECG examination showed arrythmias.

He was reffered because prior of those disorders, he complain about difficulty to see in dusk.

In the examination retinal athrophy and spiculed pigmentary degeneration was founded. The

diagnosis of this condition :

a. Mucopolysaccharidosesb. Tay-sachs diseasec. Kearns-sayre syndromed. Refsum diseasee. Bardet-Biedl syndrome

Answer D.

--good luck--

EDIT

1. A 12 year old man came to emergency room with main complain the eye lid edema since 4 days ago.

Examination result with sinusitis, eyelid edema, erythema and inflammation reaction around the orbita. VA 6/6,

pupillary reaction, and ocular motility are not disturbed. The globe are not involved. Pain on eye movement and

chemosis are absent. What is the diagnosis in this case?

A. Preseptal Cellulitis

B. Orbital Cellulitis

C. Panophthalmitis

D. Necrotizing Fasciitis

E. Aspergilosis

Page 58: Contoh Soal Baru

2. A 20 year old man came to emergency room with main complain eye lid edema and fever since 7 days.

Examination result with proptosis, chemosis, sinusitis, eyelid edema, erythema and severe inflammation. VA 6/60,

pain movement of the globe, and restriction on ocular motility .The UCG Examination normal laboratory has a

leukositosis.What is the diagnosis in this case?

A. Preseptal Cellulitis

B. Orbita l Cellulitis

C. Endofthalmitis

D. Necrotizing Fasciitis

E. Pan Opthalmitis

3. A 24 year old smoker present with painless, semiacute, progressive visual loss in his right eye. Fundus both

eyes and MRI are normal. Three months later, the affected eye has not been improved yet, and the vision has

deteoritated. Repeated MRI and lumbar puncture yield negative results. The complete blood count and assay for

antinuclea antibody, fluorescent treponemal antibody absorption, angiotensin – convertingen – enzyme, lyme

titer, vitamin B 12, and folate are normal. What is the most likely diagnosis ?

A. Multiple sclerosis (MS)

B. Tobacco – alcohol optic neuropathy

C. Posterior ischemic optic neuropathy (PION)

D. Lebel heriditary optic neuropathy (LHON)

E. Anterior ischemic optic neuropathy

4. A 10 years old boy is examined for bilateral progressive visual loss that occurred over 1 month. He has

polyuria and polydipsia and is found to have diabetes insipidus. Which of the following is the most likely

associated visual field deficit ?

A. Unilateral nasal step

B. Bilateral nasal defect respecting in the midline

C. A complete left homonymous hemianopsia

D. A bilateral temporal defect respecting the midline, denser in the inferior quadrants

E. Altitudinal

5. A 40 years old male came to policlinic with main complain pain, redness, haloes left eye. Previous history

illness : no flu like syndrome. The doctor examined and found elevated IOP on the left eye, corneal edeme, fine

KP’s, low grade cell dan flare in anterior chamber a slightly dilatated pupil , no sinechia, no vitreus cell . What is

the diagnosis in this patient ?.

A. Posner Schlossman Syndrome

B. Phacolytic Glaukoma

C. Fuchs Uveitis Syndrome

Page 59: Contoh Soal Baru

D. Voght Koyanagi Harada desease

E. Exfoliation syndrome AAO Sect 9 page 135

6. A. 22 years old female came to policlinic with main complain photopobia and decreased visual aquity both

eyes. Ophthalmological examination found visual aquity R-LE 6/20 PH( - ) , KPs (+) and snowbank on the inferior

pars plana. Retinal venous sheating at the peripheral retina and no cystoid macular edema. Patient is often

complain pain in her teeth and dentist found dental carries. What is the diagnosis in this patient ?

A. Anterior Uveitis

B. Intermediate Uveitis

C. Posterior Uveitis

D. Pan Uveitis

E. Pars Planitis AAO Sect 9 page 147

7. Young man came to RSMH with main complain diplopia, the RE deviated medialy and stabilized. From

examination we found that the patient head positionn is normal, limitation of abduction, VA equal both eyes.

Trauma history a year ago and no family history of strabismus, no proptosis and redness. Force duction test no

restriction. Which one is the weakening procedures used in strabismus surgery?

a. removal and reetachment of a medial rectus muscle , insertion is closer to its origin

b. absorbable sutures of the medial rectus muscle are placed at predetermined distance posterior

c. the muscle of the medial rectus muscle anterior to the position of the sutures is excised

d. the shortened medial rectus muscle is reattached to the globe at or near the original insertion

e. advancing the insertion of the medial rectus muscle nearer the limbus

8. A 4 months girl came with her mother with complain RE going medialy. The eye were normal at birth with good

alignment, and no history of trauma. Examination: VA exist, normal head position, limited horizontal rotation,

primary position esotropia and poor abduction. The laboratory normal. Forced duction test: restriction

What is the diagnosis of this patients?

a. congenital fibrotic exstraocular muscle

b. duane syndrome

c. graves desease

d. myasthenia gravis

e. mobius Syndroma

9. A 40 years old Man came to policlinic with a refraction disorder / astigmatism, and he has a pair of soft

contact lenses for his eyes. What is the most common adverse reaction to soft contact lenses ?

a. Giant papillary conjungtivitis (GPC)

b. Corneal neovascularization

c. Corneal hypoxia

Page 60: Contoh Soal Baru

d. Conjungtivitis

e. Keratitis AAO section 3, 197 – 199

10. A man, 23 years old wears rigid / hard gas permeable (RGP) contact lenses, in examination the position is low

riding, then inadequate tear meniscus, insufficient lens movement, thick peripheral lens profiles and abortive

reflex blink patern. What is the possible adverse reaction ?

a. 3 & 9 o’clock staining

b. Lens adhesion

c. Ptosis

d. Spectacle blur

e. Miscellaneous AAO section 3, 194

11. Bacterial infection of the orbit or periorbital soft tissues occurs from primary sources :

1. Direct spread from an adjacent sinusitis

2. Direct innoculation following trauma

3. Bacteremic spread from a distant focus

4. Direct innoculation following skin infection

AAO 7, 41

12. A girl, 3 years old came to emergency with main complain eyelid edema since 7 days. Ophthalmology

examination VODS: fixation (+), eyelid edema, pupillary reaction (+), ocular motility are not disturbed. Pain on eye

movement and chemosis are absent. Ophthalmologist diagnosed this case with preseptal cellulitis. Historically

preseptal cellulitis in infant and children was often associated with,

1. Bacteremia

2. Septicemia

3. Cutaneous infection

4. Meningitis and sinusitis

13. A 40 year old Man had basal carcinoma in the upper eyelid. He had surgery and after that he had a 40% upper

lid defect. What kind of repair needed to solve this case?

a. Primary closure with lateral canthotomy

b. Primary closure with superior cantholysis

c. Adjacent tarsoconjunctival flap and full-thickness skin graft

d. Free tarsoconjunctival graft and skin flap

e. Full thickness lower eyelid advancement flap AAO 7, 190

Page 61: Contoh Soal Baru

14. A 50 year old woman came to policlinic with main complain tearing. From examination, she had

severe involutional ectropion and involves more than the punctum. What kind of surgical generally used for this

case ?

1. horizontal resection of the lateral eyelid

2. plication of the canthal tendon

3. lateral canthoplasty

4. reinsertion of the lower eyelid retractor AAO 7, 197

15. A young man came hospital with his chief complain discomfort on his eyes, he has a dry mouth and history of

remathoid arthritis. In minor glandula salivary biopsy found inflamatory infiltrate. What is the diagnosis of this

patient?

a. Sjogen syndrome

b. SLE

c. Scleroderma

d. Reiter syndrome

e. Polymyositis

16. A young man came to a hospital with main complain fever, chills malaise. Thyroid fuction test t4 level elevated

and low RAIU. After resolution period this man got transient hipothyroidism. What is the man illness?

a. Acute thyroiditis

b. Sub acute Granulomatous Thyroiditis

c. Sub acute Lymphocitic Thyroiditis

d. Hashimoto Disease.

17. A man 20 years old came to hospital with main complain pain and red eye. Two weeks before, he had an injury

from tree of rice, from examination mix injectie was profound, paracentral ulcus with hipopion in ½ anterior

chamber. Pseudomonas aurigenosa was found from the culture, what is the effective therapy :

a. Aminoglycoside

b. Ceftazidine + Aminoglycoside

c. Cyprofloxacine

d. Imipenem and piperacilin / tazobactam

e. Ceftazidine

18. A women, 30 years old consult from surgery department with complain diplopia, enofthalmus and hyposthesia

in the distribution of infraorbital nerve. After examination a positive forced duction test, and radiographic

evidence of a fluid level in the maxillary sinus. She has diagnose Blunt trauma to the tissues of the orbit may

fracture of the fragile bony orbital floor. What is the bony orbital floor ?

a. Frontal process of the maxilla, lacrimal, orbital plate of the ethmoid, lesser wing of the sphenoid

Page 62: Contoh Soal Baru

b. Maxillary, palatine, orbital plate of the zygomatic

c. Orbital plate of the frontal, lesser wing of the sphenoid

d. Zygomatic, greater wing of the sphenoid

e. Palatine, zygomatic and maxillary

19. A father brought his 2 days old boy to an eye policlinic ,whose born with some abnormality such as strabismus,

blepharophitosis , epicantus, cataract, glaucoma, coloboma of uvea, persistent fetal vasculature,

dismorphogenesis of retina, optic nerve hipoplasia. There was no family history of this disease.What kind of drug

involved in this case?

A. Cocaine

B. Heroine

C. Barbiturat

D. Alcohol

E. Tobacco

20. A young mother went to a hospital with chief complain of ocular pain, head ache, blurred vision, rainbow

colored halo around light, nausea and vomiting. She has no history of hipertension, nor diabetes. What is the best

procedure for a good definitif diagnosis about the patient:

A. Fluoresence

B. Keratometer

C. Hertel

D. Gonioscopy

E. Tonometry

21. A 26 years old myopic man presents with a 5 day history of photopsia, small scotomas, and blurred vision both

eyes. He is recovering from a rescent flu like illnness. Examination reveals best corrected

visual acuity of 20/50 OD and 20/ 40 OS. Slit lamp examination show mild flare and cell in both

anterior chambers and mild vitreous cell in both eyes. The fundus finding are similary in both eyes with

the multifocal choroiditis and chorioretinal scars in the nasal retina. Which the following diagnoses is most likely in

this patient ?

a. Presumed ocular histoplasmosis syndrome

b. Multifocal choroiditis

c. Birdshot retinochoroidopathy

d. Acute posterior multi focal placoid pigment epitheliopathy

e. Posterior uveitis

22. A 32 year old man present without complaints for routine eye check up. On direct opthalmoscopy,

multiple patches of peripheral laticce degeneration containing multiple atroptihc retinal holes are noted in the

Page 63: Contoh Soal Baru

superior retina OD. Laticce degeneration without retinal breaks is noted inferior OS. Which of the following

statements regarding prophylactic treatment is most correct ?

a. Only the patient’s right eye should be prophylactically treated with laser photocoagulation or

cryoretinopexy

b. Both of the patient’s eyes should be propylactically treated with laser photocoagulation or

cryoretinopexy

c. If there is a prior history of retinal detachement in the left eye, propylactic laser

photocoagulation or cyroretinopexy should be considered in the right eye

d. The patient’s right eye should receive prophylactic treatment with laser photocoagulation or

cryoretinopexy prior to cataract surgery

e. Informed the patient and do nothing

23. (Refraksi, AAO 3, 239) A girl, 20 years old came to ophthalmologic policlinic, she was diagnosed with profound low

vision. Then she was consultated to rehabilitation department, for maximizing her low vision with training. She

can develop skills to make effective use of her low vision. She refers to to the gathering of visual information by

the movement of the eye or head. It is…

a. Scanning

b. Tracing

c. Spotting

d. Tracking

e. Blinking

24. (refraksi, AAO 3, 88) A girl came to ophthalmologic policlinic. She was diagnosed with simplex myopia

and compositum astigmatism, the she was given trial lenses, a spherical and cylindrical spectacle

lense at left and right eyes. When spherical and cilincdrical spectacle was used to gether at both eyes,

she fell confuse, then the doctor used spherical equivalent of the lens. It is…

a. = sphere +cylinder

2

b. = sphere+cylinder

3

c. = sphere+cylinder

4

d. =sphere+cylinder

5

25. A 29 year old man presents with a 4 week history of floaters and blurred vision in his right eye. Six

weeks ago he developed intermittent daily headaches, lympadenopathy, fever, and right sweats all of which have

improved within the last weeks. He has no history of prior medical problems, denies prior illicit drug use, is

Page 64: Contoh Soal Baru

employed as a construction worker, and is an avid hunter who frequently eats game. Visual Acuity is 20/60 OD,

and 20/20 OS, mild cell and flare is present in the right anterior chamber. The conjunctiva are notinjected, the

corneas are clear, the lens is clear, and intraocular pressures are normal. Moderately fine vitreous cell is present

in the right eye, the vitreous in the left eye is clear.fundus right eye focal retinochoroiditis in posterior pole. The

left is normal.

Which of the following diagnoses is most likely in this patient?

a. Acute retinal necrosis

b. Toxoplasma chorioretinitis

c. CMV (cytomegalovirus) retinitis

d. Lyme disease

e. Sarcoidosis p.186

26. A 27 year old man presents with a history of floaters and blurred vision OD. The patient was

immigrated from India within the last year, is currently good health, and denies any history of drug use. Two years

ago, while in India, he had a positive tuberkulin skin test and received 9 moths of prophylactic INH, Rifampin, and

Ethambutol therapy. Anterior segment is normal. The right and left fundus is show retinal neovascularization ,

Vitreous hemorrahage is noted in the inferior vitreous OD. No pars plana snowbanks are noted in either eye.

Which of the following diagnoses is most likely in this patient ?

a. Eale’s disease

b. Sarcoidosis

c. Sickle cell retinopathy

d. Cat-scratch disease

e. ARN p. 153

27. A 30 years old man came to policlinic and diagnosed with ptosis. What should we examine in this

patient?

1. vertical interpalpebral fissure height

2. margin-reflex distance

3. upper eyelid crease position

4. levator function

28. A 30 years old man came to policlinic and diagnosed with ptosis. The most commonly surgical

procedures that use in this patient are ………….

1. external levator advancement

2. frontalis muscle suspension

3. internal levator/tarsus/Muller`s muscle resection approaches

4. horizontal eyelid tightening

Page 65: Contoh Soal Baru

29. A Child 10 years old came to hospital with her mother with main complain blured vision right eye,

red eye (+), pain (+). He has trauma 2 weeks ago in examination VOD 6/60, VOS 6/6 injectie conjunctiva (+),

hifema ½ COA. He has body weight 25 kg. He must stay in hospital and get Tranexamat acid and Elevation of head

45o. How much dose of Tranexamad acid for that patient ?

a. 250 mg / day

b. 375 mg / day

c. 500 mg / day

d. 625 mg / day AAO 2 hal 453

30. A Child, 8 years came to hospital with main complain proptosis since 1 year, red eye (+), pain (+), in

examination leukokoria (+), eye like cat (+). He was diagnosed with susp retinoblastoma. What is the

predosposition factor for retinoblastoma?

a. Translocation chromoseme 13q14

b. Mutation chromosone 14

c. Delesi chromoson band long arm 13q14

d. Delesi chromosom short arm 13p14

e. Mutation chromosome 13 AAO 2 hal 253

31. A father brought his 2 days infant to eye policlinic ,whose born with some abnormality such as strabismus,

blepharoptosis , epicantus, also cataract, glaucoma, coloboma of uvea, persistent fetal vasculatur,

dismorphogenesis of retina, optic nerve hipoplasia. There was no family history of this disease. What kind of drug

involved in this case?

A. Cocaine

B. Heroine

C. Barbiturat

D. Alcohol

E. Tobacco

3. 3In the evaluation and treatment of Diabetic Retinopathy, FFA is useful:

a. To determine areas of capillary closure

b. To determine the absence of CSME prior to recommending photocoagulation

c. To determine the location of diffuse retinal leakage from incompetent retinal capillaries or IRMA

prior to photocoagulation

d. To determine the location of retinal microaneurysms prior to photocoagulation

e To determine IRMA AAO 1 2 hal 114

Page 66: Contoh Soal Baru

34. A man 32 year old with a history of AIDS and CMV retinitis initially received ganciclovir but was switched to

foscarnet 6 months later when recurrence of the CMV retinitis indicated resistance to Ganciclovir. Following

forcarnet therapy, all CMV activity in the retina resolved. The patient now presents with the fundus recurrent CMV

in the nasal retina on the left eye. His creatinine clearance is currently reduced and his white blood cell count is

critically low. What to do next?

a. Stop IV forscarnet and either give Intravitreal ganciclovir or forscarnet by injection, or use a sustained-

release ganciclovir device intravitreally

b. Repeat induction doses of foscarnet IV

c. Continue forscarnet and induction doses of ganciclovir IV

d. Add Zidavudine (AZT) to the current foscarnet therapy

e. Stop foscarnet AAO 12 hal 193

35. A 30 years old man came to emergency. From examination found proptosis, chemosis, restriction of ocular

motility, pain on movement of the globe, and fever. This patient diagnosed with orbital cellulitis that can be

caused by…….

1. dental infection

2. dacryoadenitis

3. trauma

4. endopthalmitis

AAO 7 hal 44

36. A 60 old man was cared in eye department with locally invasive tumor. He had surgery, the eye and adjacent

intraorbital tissue are removed to locally excise the lesion, leaving periorbita and part of eyelid.

What kind of type exenteration is it ?

a. Extended

b. Total

c. Subtotal

d. Perifer

e. Partial AAO 7 hal 129

37. Parents of 4 year girl complain of their daugter eyes with the occurrence of exodeviation with fatique / stress,

illness, day dreaming, drowsiness, or being scolded. Examination : exotropia manifest after fusion was disrupted

by cover testing, and the deviation was remain greater at distance than at near after a period of monocular

occlusion. We can classified this patient into………..

a. True divergence excess

Page 67: Contoh Soal Baru

b. Simulated divergence excess

c. Convergence insufficiency

d. Convergence paralysis

e. Convergence access AAO 6 p. 105 – 114

38. What is diagnosis of that patient

a. Intermitten exotropia

b. constant exotropia

c. convergence insufficiency

d. convergence paralysis

e. exophoria AAO 6 p. 105 - 114

39. A 35 years old woman complain a pain on her right eyes and blurred vision. Blurred vision happed poor

immediately with severe cephalgia. She came to ophthalmologist. She wa told that she got acute glaucoma and

need some medicine. Normally humour aqueous is produced by:

A. Pigmented epithelium layer of cilliary body

B. Nonpigmented epithelium layer of cilliary body

C. Stroma of cilliary body

D. Muscle layer of cilliary body

E. Connective layer of cilliary body AA0 10. P 17

40. A 4 years girl diagnosed with dense congenital cataract in the right eye, demonstrate exotropia 20 PD. The lens

showed opacification of the Y-sutures of the fetal nucleus. These opacities often have branches or knobs projecting

from them. Visual acuity appears normal.what is the variety of morphologic configuration of congenital cataract?

A. Sutural

B. Cerulean

C. Coronary

D. Capsular

E. Anterior polar